Sunteți pe pagina 1din 114

GENERAL EDUCATION

1. I can’t find calculator; may I use , Richard?

A. My, his B. My, yours

2. On your vacation in Palawan, please if you chance upon Arturo, please give my
warm regards.

A. Him B. Himself

3. My brother and I ran into a friend of in the park.

A. Ours B. Min

4. The cost of a new family house is quite expensive for , besides we have more
urgent family needs.

A. Them B. Us

5. Neither the principal nor the coordinators were present in the event

A. Were B. Was

6. After all day, Father finally found the time to rest.

A. Having worked B. Working

7. The empty board on the table.

A. Were left B. Was left

8. Drama to be the most interesting co-curricular activity of the year.

A. Appears B. Appear

9. The proposed project within the budget of the school.

A. Is appearing B. Is
10. It’s now two hours past his schedule; the facilitator may not come anymore, but we’ll
still be ready in case he .

A. Does B. Will come

11. We admire ________ when Cynthia speaks English fluently with


.

A. Her, us B. Him, them


12. I watched the news on television.

A. Late B. Latest

13. Let us campaign for the total ban of harmful insecticides.

A. With use B. On the use

14. When preparing ingredients, she is always careful not to hurt .

A. Herself B. Myself

15. Corrupt government officials should be ashamed of .

A. Themselves B. Ourselves

16. I thought the task was heavy for Samantha and .

A. Me B. Myself

17. Your boss isn’t working anymore, ?

A. Is he B. Isn’t he

18. The student’s request to reset the test reasonable.

A. Are B. Is

19. A number of endangered species living underwater now extinct because of illegal
hunting and dynamite fishing.

A. Is B. Are
20. Technology dramatically in the twenty-first century.

A. Has improved B. Did improved

21. If I the same problems you had as a child, I might not have succeeded in life as
well as you have.

A. Have B. Had

22. You finished eating before she arrived.

A. Have B. Will have

23. If I could speak English well, I next year studying in the States.

A. Would have spent B. Would spend

24. The principal has issued the memorandum before she _______ the student’s request.
A. Received B. Will be receiving

25. Had I studied very well, I _______ rewarded with a tour around El Nido.

A. Would have been B. Would be

26. The means of transportation dramatically since the end of the 20th century.

A. Has changed B. Have changed

27. The tribulations that I faced were for me to overcome.

A. So much B. Too much

28. Joan lives Santillan Road.

A. In B. On

29. Maggie will be ready to leave about thirty minutes.

A. In B. At
30. Since he met his new girlfriend, Mico never seems to be home.

A. In B. At

31. Jeremy responded to his mother’s demands throwing a tantrum.

A. With B. By

32. I think Carmen spent the entire afternoon the phone.

A. In B. On

33. The nearest post office is on .

A. Twenty Second Street


B. Twenty-second Street

34. Have you ever been to my ?

A. Brother-in-law’s repair shop, Gus’s Garage


B. Brothers-in-law’s repair shop, Gus’s Garage

35. Please read the chapter .

A. “Filing Your Income Tax Returns.”


B. “Filing your Income Tax Returns.”

36. New students at the shop should bring .


A. Hammers, wrenches, and screwdrivers
B. Hammers, wrenches and screwdrivers

37. Mrs. Navarro said that .

A. the Chemistry II exam will be next Wednesday.


B. “The chemistry II exam will be next Wednesday.”

38. “It is only with the heart that one can see rightly.” This line means
.

A. Only the heart can see the beautiful things in life.


B. The heart is always correct about everything.
39. The man in the mirror looks:

A. Strange B. Strangely

40. Several factors must be considered in order to make a sound .

A. Query B. Decision

41. Those are probably the blankets in the store.

A. Most fancy B. Fanciest

42. Lolo Jorge is really man.

A. A sweet old B. An old sweet

43. are activities which a heart patient must forego.

A. Heavy smoking and overeating


B. Smoking heavy and to overeat

44. In the small town of Tubao, to the green hills and plains.

A. One can quickly walk


B. Where one can quickly walk

45. One method of ending discrimination in business and industry is


.

A. To demand quotas to be met by employers


B. To demand that employers must meet quotas

46. Your findings are impertinent to the results of this investigation. Select the word which
means the same as the underlined word.
A. Important B. Irrelevant

47. Several elements must be transmuted to see the real implications of this event. Select the
word which means the same as the underlined word.

A. Summed B. Changed
48. You should be ashamed of yourself for having such carnal desires. Select the word
which means the same as the underlined word.

A. Worldly B. Bold

49. The President showed great aplomb while dealing with the reporters. Select the word
which means the same as the underlined word.

A. Nervousness B. Composure

50. The courage of the son who saved his father from a raging fire is worth

A. Emulating B. Watching

51. His apocalyptic views seemed to doom the human race. This is to be taken as .

A. Apathetic B. Prophetic

52. Our guest of honor was a loquacious speaker. This means .

A. Reserved B. Verbose

53. Except for the profanity, I like the story. What is the story about?

A. Obscenities B. Shamelessness

54. Which word/phrase is nearest in meaning to the italicized word? He has been very
feeble since his illness?

A. Weak B. Unwell

55. Which word/phrase is nearest in meaning to the italicized word? Jose was contemptuous
of the effects of the efforts of his rival to show greater ability.

A. Scornful B. Appreciative

56. She has a penchant for red accessories. This indicates .

A. Dislike B. Fondness
57. The word “baduy” means

A. Awkward looking B. Low profile


58. That hard work has it rewards appears to be the of the story

A. Moral B. Morale

59. Archimedes shouted “Eureka!” What did we take this to mean?

A. Enjoyment B. Discovery

60. What figure of speech is the following: “He was helpless as a child.”

A. Simile B. Metaphor

61. “…It droppeth as a gentle rain from heaven” is an example of this figure of speech

A. Simile B. Metaphor

62. For the synecdoche figurative “My mother has a green thumb. I can’t wait to show you her
garden.” What is the equivalent meaning?

A. She cultivates a garden


B. She is good at gathering

63. What figure of speech is: “O wild west wind!”

A. Apostrophe B. Hyperbole

64. What is the figure of speech in the sentence: “The sun was a ball of fire over the
mountain.”

A. Simile B. Metaphor

65. “She is my sunshine” is an example of

A. Simile B. Metaphor

66. “My head is bloody, but unbowed” is an example of which figure of speech?

A. Hyperbole B. Metaphor
67. “Sturdy and strong, the Filipinos are like the molave” is

A. Simile B. Metaphor

68. What figure of speech is: “A tree whose hungry mouth is pressed against the earth’s
sweet flowing breasts.”

A. Personification B. Hyperbole

69. “He was like a snake in the night.” What figure of speech is present?
A. Simile B. Metaphor

70. News travels as fast as the wind. This statement is a/an:

A. Simile B. Metaphor

71. What figure of speech is used in the line “I wondered lonely as a cloud…”

A. Simile B. Metaphor

72. “The Lord is my shepherd” uses this figure of speech.

A. Simile B. Metaphor

73. “He is the black sheep in the family.” Which figure of speech is used?

A. Metaphor B. Hyperbole

74. What figure of speech is employed when the author states, “O souls and spirits of the
martyred brave, arise?”

A. Apostrophe B. Metonymy

75. October 10 is a “red letter day” for the Chinese. Classify the quoted expression.

A. Idiom B. Metaphor

76. “A man is known by the company he keeps.” This is an example of a


.

A. Proverb B. Poem
77. Shakespeare once said, “My honor is my life, both grow as one, take honor from me and my
life is gone.” What is the correct interpretation for this?

A. His life is determined by his honor


B. He values honor as much as his life

78. “The Lord is my Shepherd; I shall not want…” is an expression of one’s


.

A. Blind ignorance
B. Total submission to God

79. The line from the Koran, “Woe to every backbiter,” talks of .

A. Sorrow for those who oppose other’s opinion


B. Misery for those who talk ill of others

80. What season is it, with the line, “He ruffles every lily-pond? Where blossoms kiss and
part.”

A. Spring B. Autumn
81. The hostages are free. They paid their way. What does this statement tell us?

A. They negotiated peacefully.


B. After paying ransom they were released.

82. She wrote a “first-rate” report. What does it say about the report?

A. Excellent B. Run-of-the-mill

83. Our commitments can develop us or they can destroy us, but either way, they define us.
Choose the line below that has the same meaning.

A. Our lives are shaped by what we are committed to


B. We must make full commitment to wordly goals

84. The Bible tells us, “There is a right time and a right way to do everything.”

A. Plan well what you say and do.


B. Say and do things anytime.
85. Thomas Carlyle said “The man without a purpose is like a ship without a rudder-a waif…” It
could be interpreted as:

A. There should be a strong driving force in one’s life.


B. There should be sufficient energy in one’s life.

86. Shakespeare once said: But love is blind and lovers cannot see; the pretty follies that they
themselves commit. What do the underlined words mean?

A. Careless acts B. Nonsense words

87. What does the expression “love begets love” mean?

A. If you offer love, you will be loved in return.


B. Love is offered, not waiting for love in return.

88. Imprisoned for 27 years, Nelson Mandela realizes dream of a free and non-racial South
Africa by forgiving his oppressors and negotiating in good faith for the peaceful transition
to democracy. Nelson Mandela demonstrated his trait of .

A. Friendliness B. Forgiving

89. The Senator approached the podium with his spiel. His hands were shaking and he was
sweating tremendously. He experienced:

A. Nausea B. Stage fright

90. What role of the women is displayed by the lines in Luis Dato’s Spouse: “She holds no joys
beyond the day’s tomorrow, finds no world beyond her love’s embrace, looks upon the form
behind the furrow, her mind, motion, time, and space?
A. Work partner B. House wife

91. “I have always been financially dependent by continuing to teach even when I got
married. At times though when the cares of life weighed heavily on me, I would think of
my mother and her example of making the most of what she had. She did her household
chores methodically, tutored us in our lessons and gave modest contributions to worthy
causes.” What adjective best describes the mother?

A. Sacrificing B. Dutiful 10
92. What became prophetically true in France when Edwin Markham saw the French
peasant as “The man with the Hoe”, a plundered slave under the rule of kings and
princesses?

A. French revolution
B. Napoleon’s imperial rule

93. Select the most appropriate heading for the paragraph: Before he has any actual
knowledge of war, Stephen Crane wrote the American Classic “The Red Badge of
Courage”, which made him an overnight success. He wrote “The Open Boat” as a result
of shipwreck he experienced on his way to Cuba to report a war in progress there.”

A. The Open Boat


B. Stephen’s Writings

94. In William Shakespeare’s “Merchant of Venice”, Portia, the judge, said, “The quality of
mercy is not strained; it droppeth as a gentle rain from heaven upon the place
beneath…It’s mightiest…and one is likened to God when mercy seasons justice.” The
main thought of the excerpt is that

A. Mercy is limited in extent


B. Mercy is spontaneous and freely given

94. “My concept of inner peace came from my mother’s daily activities which I now recall
with fondness and awe. She was a full-time housewife wholly dependent on my father’s
monthly salary. How she made both ends meet, guided us in our studies and did small acts
of charity on the side was beyond me.” Based on the recount, the mother’s financial
resources were:

A. Limited B. Enough

96. “I am a retired a public school teacher. As a teacher, I was branded as a terror in school. The
pupils dreaded the day they would enter my class. Little did they know that behind my
unpopular façade was a heart full of compassion. But how did I learn this moniker? I did
not tolerate dirty pupils in my class. I wanted them to know that cleanliness of body
was good for their health. I inspected their teeth, nails, footwear, handkerchiefs, clothes,
ears, noses and hair.” Based on the recount, the teacher values .

A. Cleanliness B. Study
97. Close relatives in an extended family are provided food and accommodation and
in turn help in the household activities making the relationship .
A. Reciprocal B. Friendly

98. Though nothing can bring back the hour of splendor in the grass, of glory in the flower,
we will grieve not; rather find strength in what remains behind. The passage expresses

A. Hope B. Despair

99. Who has always been called the Morning Star of English literature?

A. Shakespeare B. Geoffrey Chaucer

100. “To be or not to be” is the beginning of the soliloquy of .

A. Macbeth B. Hamlet

101. Which play by William Shakespeare features to ill-starred lovers who come from
warring families?

A. Rome and Juliet


B. Anthony and Cleopatra

102. The Rubaiyat has the theme:

A. Grasping pleasure while you can


B. Creating your own world and beautify it

103. Who was the foremost French short story writer?

A. Anatole France
B. De Maupassant

104. What are the greatest lyric poems in the literature of the world?

A. The Psalms of Praises


B. The Psalms of King David

105. The pen name of Samuel Clemens is .

A. Mark Twain B. George Elliot


106. Filipino migrant writer whose fiction stories reflect the Filipino’s concept of American
culture?

A. Bienvenido Santos
B. Ambeth Ocampo

107. This Filipino writer in English used Hispanic-Filipino culture and traditions in his fiction
works.

A. Nick Joaquin B. Jose Garcia Villa


108. What is a dramatic presentation that originated from the traditional armed encounter
between the Christians and Muslim Filipinos?

A. Moro-moro
B. Comedia de Lara y Espana

109. One famous Japanese poem is known as haiku. It consists of

A. Three lines totaling 17 syllables with nature as topic


B. Four lines that rhyme with love as topic

110. If a story is made out of an exchange of letters, it will be called literature.

A. Epistolary B. Episodic

111. Among the kinds of listening, which is the type that most students do in the classroom?

A. For information and ideas


B. For enjoyment

112. In the speech progress, which is concerned with amplifying and enriching the voice,
using human amplifiers such as the nose, windpipe and chest?

A. Resonation B. Phonation

113. Of the seven continents comprising a continuous land mass surrounded by big
bodies of water, the biggest (30.1 percent of the earth) is .

A. Africa B. Asia
114. Baghdad is to Iraq and is to Syria.

A. Jordan B. Damascus

115. Hers is … “the face that launched a thousand ship”

A. Helen of Troy
B. Princess of the Nile

116. Where did the ancient civilization of China exist?

A. Nile River B. Yellow River

117. Analects which are short and witty sayings that treat moral values and good human
relation, are attributed to this Chinese writer:

A. Confucius B. Lao-Tzu

118. Indian sacred hymns are enshrined in this compendium

A. Rig Veda B. Mahabharata


119. This person is the world’s first mythical geographer, and he was recognized due to
his vivid descriptions of lands and people encountered by his hero Ulysses.

A. Virgil B. Homer

120. Egyptian literature is identified as literature.

A. Mediterranean B. African

121. The Philippines lies in the ________, an area where volcanoes are active.

A. Wheel of fire B. Ring of fire

122. Before migrants crossed the seas from the southern Philippines, there were already
aboriginal settlers in the islands. Who were these?

A. Negritos B. Malayans
123. What is considered the earliest form of writing in the Philippines?

A. Abakada B. Alibata

124. What is one of the man-made wonder of the Philippines?

A. Rice terraces B. Manila Bay

125. Who first introduced the Islamic religion to the Philippines?

A. Mukdum B. Rajah Baguinda

126. The late president Ferdinand Marcos placed the Philippines under Martial Law through
which of the following laws?

A. Proclamation 1081
B. Kautusang Tagapagpaganap blg. 25 S. 1974

127. In July 1901, Isabelo delos Reyes founded the first labour union in the country. What was
its name?

A. Union Obrero Democratica


B. Union Trabajadores de Filipinos

128. This religious institution is the only living remnant of the Philippine Revolution of 1896?

A. Philippine Independent Church


B. United Church of the Philippines

129. What building was the only one left intact after the destruction of Intramuros during
the Battle of Manila?
A. San Agustin Church
B. Manila Cathedral

130. This person served as the brains behind the “arena theater”.

A. Montano B. Avellana

131. She wrote the 1st Filipino modern English language short story, Dead Star, published in
the Philippine Herald in 1925.

A. Paz Marquez Benitez


B. Ophelia Dimalanta
132. Which among the following pertains to the passionate Filipino dance that makes use of a
fan or handkerchief to illustrate a couple in romance?

A. bo B. Carinosa

133. Famous ballerina who brought ballet to children of various economic levels?

A. Lea Salonga B. Liza Macuja

134. Angono, Rizal boasts a famous composer who became a National Artist. Who is this
person?

A. San Pedro B. Cayabyab

135. Who is Quijano de Martin in Philippine Literature?

A. Nick Joaquin
B. Jose Garcia Villa

136. Who was the literary figure known as “Huseng Sisiw”?

A. Jose dela Cruz


B. Juan Matapang Cruz

137. The following were among the five ships involved in Ferdinand Magellan’s voyage,
except:

A. Trinidad B. Cartagena

138. What was the name of forced labor imposed during the Spanish regime?

A. Bandala B. Polo

139. Through the Galleon Trade, the Philippines had extended contacts with .

A. Mexico B. Spice Islands

140. What economic policy in 16th century Europe influenced Spain’s expansion is policy to
discover new lands?
A. Mercantilism B. Colonialism 16
141. 1565, Legaspi concluded a blood compact with the chief of Bohol. Who is referred to?

A. In Lakandula B. Sikatuna

142. Which of the following was the first to happen?

A. The Philippines was ceded to the US by the Treaty of Paris


B. Guerilla warfare against the US was initiated

143. Which religious missionaries first arrived in the Philippines?

A. Augustinians B. Franciscans

144. The Spanish mestizo priest was the pioneering leader of the native secular clergy in the
Secularization Movement of 1861.

A. Father Pedro Pelaez


B. Father Gregorio Aglipay

145. Governor-General Narciso Claveria was responsible for the


.

A. Use of Spanish surname by Filipinos


B. Establishment of the Tobacco Monopoly

146. During the Hispanic era, native and Chinese male mestizos 16-60 years old are
obligated to do forced labor for forty days. How can exemption be made?

A. Paying the bandala


B. Paying the falla (exemption fee)

147. Which Spanish policy provided the resettlement of Filipino communities to


form town centers of cabeceras?

A. Reduccion B. Bandala

148. The last Spanish Governor-General of the Philippines

A. Basilio Agustin
B. Diego de los Rios

17
149. During the Spanish era, who was the revolutionary leader who waged a long war against
the government because it did not allow proper burial for his brother?

A. Macario Sakay
B. Francisco Dagohoy
150. Even after General Malvar’s surrender, there was a revolutionary leader who refused to
surrender to the Americans. Instead, he declared himself President and
Commander-in-chief of the supreme Government of the Tagalog Archipelago. Who
was this person?

A. Macario Sakay
B. Artemio Ricarte

151. Gregoria de Jesus saw her husband ______ charged for treason by fellow Filipinos who
have formed the Philippine revolutionary force to fight Spain.

A. Andres Bonifacio
B. Jose Rizal

152. During the June 12, 1898 declaration of Philippine Independence, a band played the
Marcha Nacional Filipino. What band was this?

A. Malabon Band
B. San Francisco del Monte Band

153. Marcelo H. del Pilar condemned the hidden control and domination by Spanish religious
priests over the colonial government using this term.

A. Complace B. Frailocracia

154. He became the leader of the Magdalo faction.

A. Mariano Alvarez
B. Baldomero Aguinaldo

155. Who was the founder and editor of the newspaper “La Independencia”?

A. Antonio Luna
B. Graciano Lopez Jaena 18
156. Who was the orator of the Reform Movement and the first editor of La Solidaridad?

A. Marcelo del Pilar


B. Graciano Lopez Jaena

157. What law passed on August 1909 by the U.S. Congress established a partial free trade
in the Philippines?

A. Payne-Aldrich Act
B. Underwood-Simmons Act

158. Which law set a full-tree trade policy abolishing the quota limitations on Philippine exports
to the United States?
A. Payne-Aldrich Act
B. Underwood-Simmons Act

159. Which law enacted by the U.S. Congress in 1916 established a bicameral legislature in the
Philippines with the senator (upper house) and House of Representative (lower house)
to be composed of elected Filipinos?

A. Cooper Act B. Jones Law

160. What contain the concrete promise of the Americans to Filipinos independence “as
soon as stable government can be established”?

A. Preamble of Jones Law


B. Preamble of the Commonwealth Government

161. Who was the American president who proclaimed the Benevolent Assimilation policy
during the American colonial years of the Philippines?

A. William Mckinley
B. Franklin Roosevelt

162. Who was among the last Filipino generals who fought the Americans and established the
so-called “Tagalog Republic?”

A. Macario Sakay
B. Miguel Malvar
163. In Philippine history, who was known as the “The Great Dissenter?”

A. Manuel Roxas B. Claro M. Recto

164. Who among the following Filipino statesmen was the most vocal to speak against the
country’s foreign policy of close association with Americans?

A. Claro M. Recto
B. Cayetano Arellano

165. He is considered as the “Father of the Local Government Code”.

A. Jovito Salonga
B. Aquilino Pimentel

166. Who is considered as the father of Iloco Literature?

A. Pedro Bucaneg
B. Pedro Paterno

167. Where did the folk song “Atin Cu Pong Singsing” originate?

A. Ilocanos B. Pampangos
168. One instance of taking away the life of another person without due process is .

A. Salvaging B. Euthanasia

169. How many days are needed after which an enrolled bill becomes a law?

A. 90 days B. 30 days

170. Which policy of the state provides preferential attention to the welfare of the less fortunate
members of Philippine society?

A. Distributive justice B. Social justice


171. Participation in governance, including the right to vote and seek public office is secured
within the citizenry’s .

A. Political rights
B. Right of suffrage

172. Which best describes the division of the Legislature in to the Senate and the House of
Representatives?

A. Bipartisanship
B. Bicameralism

173. Which department has the authority to make laws and to alter them when needed?

A. Legislative B. Administrative

174. If this principle is applied well, no branch of government will be too powerful or abusive.

A. Check and balance


B. Impeachment

175. Which of the following is a way for the judiciary to check on the executive branch of
government?

A. Determining the salary of the president and vice president


B. Declaring an act of the president unconstitutional

176. Which is the fundamental law of the Philippines?

A. The Philippine Constitution


B. The Bill of Rights

177. Which condition contradicts the social justice mandate in the 1987 Constitution?

A. Creation of economic opportunities for all


B. Sense of dependency among citizens
178. Which refers to the act of the President to stay the execution of a convict?
21
A. Pardon B. Reprieve
179. Who has the power to declare the existence of a state of war?

A. Congress B. President

180. A losing senatorial candidate may file a protest against the winner. What body will
assume jurisdiction over the case?

A. COMELEC B. Electoral Tribunal

181. How is the crime of rape classified?

A. Malicious mischief
B. Heinous

182. All Filipino citizens have the right to vote and to be voted upon as a government official.
What is this constitutional right called?

A. Suffrage B. Electoral right

183. Any individual has the right to question why he is being arrested, and to summon his
accuses to court so that due process may be performed. This right is embodied in which
of the following?

A. Res ipsa loquitor


B. Habeas corpus

184. The tax required to be paid annually by all adult citizens of the Philippines is the
.

A. Community tax B. Income tax

185. Which among the following represents the political process involved in amending or
revising the Philippine constitution to adapt to the present Philippine situation?

A. Constitutional convention
B. Charter change

186. If the social system is based on meritocracy, which is a possible effect?

A. Rule by wealthy and powerful


B. Leadership by the people of talent
187. In the decade of the 70s, one clamor of the activists who staged street demonstration on the
streets was “Down with the oligarchs!” What did they accuse government of as
suggested by the underlined word? Government was:

A. A rule of the few rich


B. Elitist

188. With the Batasang Pambansa performing legislative and executive powers in the
Marcos regime, which form of government was implemented?

A. Presidential B. Parliamentary

189. To which type of political system do we belong?

A. Democracy B. Colonialism

190. In which form of government does power or authority reside in a few persons who govern
for their own interest?

A. Monarchy B. Oligarchy

191. Which of the following represents the smallest subunit of government in the Philippines at
present?

A. Barangay B. Sitio

192. The Philippine government is divided into three branches: executive, legislative and
judiciary. Which among the following doctrines best describes the model of governance
in the Philippines?

A. Division of labor
B. Separation of powers

193. The right invoked in the “writ of habeas data.” Is

A. Right to search for information


B. Right to privacy of information

194. The fundamental right invoked by filing the writ of amparo is


.

A. Right to due process


B. Right to life, liberty, and security
195. Which concept best showcases social and environmental justice interconnection?

A. Industrialized countries’ toxic disposed to poorer countries


B. Desertification of verdant regions

196. How do you call the tax imposed on all employed and practicing professionals?

A. Income tax B. Community tax

197. Which of the environmental law requires industries to install anti- pollution devices and
bans the use of incinerators?

A. Clean Air Act


B. Environmental Security Act

198. Based on the poem, “Sa Aking Mga Kababata” written by Jose Rizal as a young poet,
what was he advocating?

A. The use of both the native tongue and multilingual languages


B. The use of one’s native tongue (mother tongue)

199. In his “La Indolencia del Filipino,” Rizal’s thinking was the rich natural resources of the
Philippines and the favourable climate were not a blessing after all for the Filipino. Which
explains this?

The rich natural resources and the favourable climate of the country
.

A. Made the Filipinos compere against one another


B. Encouraged the “get-by” mentality

200. Which among the following is the main theme of the El Filibusterismo?

A. Curing the social cancer


B. Ideal means of achieving social reform

201. What was the title of Rizal’s work considered to be his crowning glory as a young poet?

A. Mi Ultimo Adios
B. A La Juventad Filipina
202. What is the art and science that deals with the morality of human acts?

A. Ethics B. Logic

203. In Rizal’s time which among the following was pressing issues in Philippines society
that were presented in Noli Me Tangere?

A. Abuses against Filipino women


B. Abuses of friars

204. Ethical conduct governs social research. Which among the following are not related to
ethics/morality in research?

A. Erroneous data
B. Voluntary participation of respondents

205. Whose view gives prominence to faith in understanding the existence of God?

A. St. Augustine B. St. John

206. Reason must be used in understanding the existence of God. Who advocated this
philosophy?

A. St. John B. St. Thomas Aquinas


207. This field deals with the study of how human beings behave.

A. Psychology B. Ethics

208. If principles and theories of human behavior were to applied to teaching and
learning, the field will be called

A. Educational philosophy
B. Educational psychology

209. The father of modern psychology

A. Sigmund Freud
B. Wilhelm Wundt
210. Which of the following is considered as the lowest form of learning?

A. Perceiving B. Responding

211. Which statement on human intelligence is correct?

A. It is verbal ability and abstract reasoning


B. It consists of multiple intelligences

212. Of the following, which is an example of natural altered state of awareness?

A. Sedation B. Sleep

213. What is Ausubel’s belief on the learning of the children?

A. Learn best when given ample opportunity for autonomous discovery


B. Children can be taught concepts in similar ways that adults
are taught as long as the materials is meaningful

214. A behavioural psychologist would probably say that:

A. The mind is most important when analyzing behavior


B. We act based on rewards and punishments received

215. Which characterizes psychologically healthy people?

A. Concerned with goals and beyond themselves


B. Feels superior over others

216. Which of the following is not true of mentally retarded children?

A. Grasps things easily


B. Short attention span
217. If a person claims he has a strong sense of self-efficacy, which is not reflective of
him/her?

A. View challenging problems are tasked to be mastered


B. Avoid challenging tasks
218. Which statement is not true of human growth and development?

A. Individual undergoes similar stages of development


B. The rate of development is the same in everyone

219. The tendency to imitate elders is very strong among children. Teachers should therefore
be very good

A. Role models B. Disciplinarians

220. Which term refers to the social aspects of sex or to socially defined roles and
expectations that are associated with sex?

A. Social differentiation
B. Gender

221. Which need is highlighted by the statement “Children cannot learn with empty
stomach”?

A. Physiological B. Esteem

222. Which idea contradicts the concept of free will?

A. Evolution theory
B. Biological determinism

223. Which human need is considered basic?

A. Security B. Wealth

224. Which among the following represents a detrimental aspect of Philippine culture
and society?

A. Palabra de honor
B. Ningas cogon

225. Material culture refers to the tangible and concrete objects produced by man. Which are
examples of material culture?

A. Sinulog celebrations
B. Stone walls of Tasadays
226. What is the term for the unpleasant sensation of frustration when a traditional bound
Filipino goes to the U.S. and observes the open display of love and emotions?
A. Culture shock
B. Culture phobia

227. When teachers brand as “pilosopo” students who ask questions, they communicate
the negative sociological principle that

A. Asking questions in class is a sign of inattentiveness


B. Passive and conformity are worth perpetuating

228. Which of the following best describes the concept of society?

A. The same as culture or nation


B. Organized interaction of people sharing land and culture

229. The primary biological limitation of natural family planning is that it is:

A. Goes against the tenets of the Catholic church


B. Only suitable for females with regular periods

230. What is the bet reason for holism in art education?

A. Everyone is a potential artist with talents of his or her own


B. Art means the beautiful in human creativity

231. Art is the handmaid of religion. Why do Jews and Muslims allow signs and symbols
and not human beings in religious art?

A. People might worship images themselves


B. Signs and symbols display hidden mysteries

232. Due to close family ties, Filipino families are generally

A. Nuclear B. Extended

233. If the family is matriarchal, who will the leader be?

A. Mother B. Grandmother
234. In what sense is the Filipino likened to a bamboo?

A. He is flexible B. He is sturdy

235. The Filipino’s indomitable spirit is shown in his .

A. Self-reliance B. Ability to survive

236. On which Filipino trait does Gawad Kalinga build on?

A. Bayanihan spirit
B. Joy and humor
237. Out of used illustration boards, students came up with cute and functional bags for
school children. Which Filipino trait is exhibited?

A. Creativity B. Adaptability

238. The Filipinos are deeply religious; however, sometimes this religiosity instills the
negative attitude of:

A. Resignation, which leads to doing nothing to improve one’s


condition
B. Total surrender to God in times of helplessness is best

239. Then Philippines is an example of a predominantly country in the world.

A. Christian B. Muslim

240. Which of the following types of societies came first during societal evolution?

A. Horticultural B. Industrial

241. In a sociological sense, earning a 1.0 Grade Weighted Average (GWA) makes you a
deviant because:

A. You could only have gotten that CGWA by cheating


B. A GWA of 10 is not the norm
242. What is the underlying theory of the belief that the DepEd and other stakeholders should
exert deliberate efforts to construct a satisfying culture for education?

A. Cultural Revitalization Theory


B. Anarchistic Theory

243. Of the following remarks from workers, which deserve a higher wage owing to effective
speech what increases work efficiency?

A. I have strong hands, willingness and honesty in work


B. I like people and listen to respond their opinions and needs

244. What problem can result owing to bias among parents and the youth towards manual or
technical occupations, e.g. Masons, mechanics, plumbers, etc?

A. White collar mania


B. Blue collar mania

245. Which among the following are qualities of an authoritarian parent?

A. High level of control and a low level of warmth


B. High level of both control and warmth

246. An individual who can find minute differences between small objects possesses which of
the following?
A. A high difference threshold
B. A low difference threshold

247. Among the developing Asian nations, which of the following is widely promoted to be the
better solution to poverty and unemployment?

A. Urbanization
B. Industrialization

248. The method of classical conditioning begins with:

A. The unconditioned stimulus and response


B. The conditioned stimulus and response
249. What does “capital” in economics refer to?

A. Investment and loss computed


B. Money/machines invested to transact business

250. What results when the government spends more than what it collects in the form of
taxes?

A. Fiscal overspending
B. Fiscal deficit

251. One of the major functions of taxes is to finance real government expenditures. This
function can simply be stated as to:

A. Promote development
B. Raise revenues

252. What is the type of cooperative which promoted thrift among members and
creates funds in order to be able to grant loans?

A. Service B. Credit

253. Which term refers to the measures undertaken by the government which would remove
anything that hinders the entry of foreign products, services and capital between
countries?

A. Liberalization
B. Globalization

254. What do we call the sum of money collected for our use of a road, bridge and
highway?

A. Tax B. Toll

255. Which term refers to duties payable on goods, whether imported or exported?
A. Subsidiary B. Tariff

256. When on controls the supply/production of goods, this implies

A. Inflation B. Monopoly
257. To stimulate the nation’s economy and increase employment, former President
Diosdado Macapagal:

A. Abolished government repressive controls


B. Standardized the salaries of government employees

258. Which of the following best described the goal of agrarian reform?

A. Abolish share tenancy in favor of less tenancy


B. Distribute all arable lands to the landless

259. Which task does an operating system for a computer perform?

A. Data compression
B. Memory management

260. Which one links the computers to base computer making it possible to share software
materials?

A. Local Area Network


Internet B.

261. Which enables the computer to communicate with other computers over the telephone?

A. Modem B. Network

262. When is it most important to scan a computer’s drives for viruses? Immediately after
.

A. Downloading a text file from a government website


B. Installing software from a disk created by a friend

263. Like the typewriter, the computer has a .

A. Screen B. Keyboard

264. Which refers to a single phrase or word that tells the computer to do something with a
program or file?

A. Command
B. Computer program
265. A secret word that must be entered into the computer before a person is allowed to get or
change information is called :

A. Password B. Input
266. In writing a letter, which productivity tool is used?

A. Publisher B. Word processing

267. Which application program provides users with a means to organize and present
information through the use of text, numbers, graphs, sounds and visual images?

A. Publisher B. Multimedia

268. What symbol is used to make the text bold?

A. Ctrl + B B. Ctrl + V

269. means the graphics will be displayed more than once.

A. Tiling B. Tagging

270. Which keyboard information is used to open a document in Windows operating systems?

A. Ctrl + D B. Ctrl + O

271. Which is the brain of the computer?

A. RAM B. CPU

272. The acronym HTML means Hyper Text Mark-Up

A. Language B. Label

273. Three-dimensional image reproduced from a pattern of interference is called

A. Hologram B. Network
274. Activities connected by a computer system are described as

A. Network B. Online

275. Which among these types of graphs best illustrates the progress in academic grades
over four quarters?

A. Line graph B. Bar graph

276. What illustrates a particular data series through rectangles?

A. Line graphs B. Bar graph

277. A social network services available through the computer is .


A. I-pad B. Facebook

278. Which program application provides users with a means to organize and present
information through the use of text, numbers, graphs, sounds, and visual images?

A. Multimedia B. Spreadsheet

279. What refers to a computer program?

A. Software B. Terminal

280. The information highway is called .

A. Internet
B. Information technology

281. In the phrase, “six times a number increased by seven raised to the fifth power, the
exponent is .

A. 5 B. 7

282. The island of Luzon is estimated to be 100, 000 square kilometers. In exponential from, it
can be expressed as .
105

A. 1 x 106 B.
283. How many prime numbers are there between 1 and 100?

A. 25 B. 24

284. What are the prime factors of 120?

A. 2x3x4x5 B. 2x2x2x3x5

285. What are the prime factors of 56?

A. 7, 4, 2 B. 7, 2, 2, 2

286. The least common multiple of 2, 3, and 4 is .

A. 12 B. 24

287. What is the LCM of 5, 2, and 7?

A. 35 B. 70

288. What are the least common multiple of 12 and 60?

A. 120 B. 360
289. The largest common factor of two or more numbers is called?

A. Greatest common factor


B. Prime factor

290. The greatest common factor of 22, 15, 7 is A. 1 B. 2

291. Among the given decimals, which is equivalent to 9%

A. 0.09 B. 0.9

292. What percent if 75 is 15?

A. 20% B. 30%

293. Total amount after adding 8% interest for 3 months of P 6, 000?

A. P 6, 120 B. P 10, 500


294. Iah left her house at 2 PM travelling at a constant speed. At 4 PM, her brother Prince drove
at 70 kph to catch up to her. If Mia was overtaken at 9 PM, at what speed was she
travelling?

A. 40 kilometers per hour


B. 50 kilometers per hour

295. The probability of getting a 2 after rolling a fair die is A. 1/6 B.

296. If a die is rolled, what is the probability of getting a number divisible by 2?

A. 1/2 B. 1/3

297. A ball is drawn at random from a box containing 6 red balls, 4 white balls, and 5 blue balls.
Find the probability that is it is blue.

A. 1/3 B. 2/5

298. In how many ways can 5 girls be seated in a row of 5 seats?

A. 120 B. 100

299. A recipe calls for 2 cups of milk for every 7 cups of flour. A chef will use 28 cups of flour,
how many cups of milk must he have?

A. 8 B. 14

300. In a university, the ratio of female professors to the male professors is 8:5. If there are
288 female professors, how many are male professors?
A. 180 B. 120

301. How many seconds are there in a 24-hour day?

A. 86, 400 B. 84, 600


302. What number must be subtracted from both numerator and denominator of the
fraction 11/23 to give a fraction whose values is 2/ 5?

A. 4 B. 3

303. The sum of three consecutive integers is 123. What are the integers?

A. 40,41,42 B. 39,40,41

304. Which of the following is a product of 13 and an integer?

A. 1326 B. 1323

305. Simplify 6 – (3 – (4) + 11 + 8)

A. 20 B. -20

306. Simplify 5 – (2 – (-4)+ 11 – 8)

A. 4 B. -4
307. Simplify: x – 2y (x – 8y) – (-6xy + 7x)

A. -6x + 3xy + 24y2 B. 6x – 3xy – 24y2


308. Simplify: (8x – 24) / (2x2 – x – 15)

A. 8 / (2x + 5) B. 4 / (x – 13)

309. Subtract 5a-2b from the sum of 7a+5b and a+b A. 3a + 14b

B. -3a + 14b

310. If 50% of x is 20, what is 20% of x?

A. 16 B. 8

311. The mean of an x and negative five is ten. What is the value of x?

A. 25 B. 20
312. Janus took 6 tests in Art. She got an average of 76% on quizzes 1 and 2 while she got an
average of 85% on quizzes 3 to 6. What is her average score for the six tests?

A. 82% B. 82.5%
313. The following are Mathematics test scores: 10, 15, 12, 18, 16, 24, 12, 19,
14. What is the median score?

A. 14 B. 15

314. The scores of five students in an English exam are as follows: 80, 75, 60, 95, and 100.
What is the average score?

A. 82 B. 83

315. In an English test, eight students obtained the following scores: 12, 10, 13, 11, 15, 20,
19, 17. What is the median score?

A. 15.5 B. 14

316. The following measures are affected by outliers, except

A. Median B. Mean

317. Given the test scores 10, 12, 14, 10, 12, 11, 10, 11, 12, 13, 10, 12, 12, 11,
10, the distribution is

A. Bimodal B. Unimodal

318. The sum of the sides of a polygon is the .

A. Perimeter B. Area

319. All right angles are

A. Equal B. Supplementary

320. The altitudes of the base of a triangle are 24 meters, and 5 meters respectively. What
is its area?

A. 60 square meters
B. 50 square meters 38
321. Kalian nagsimula ang pagkakaraoon ng modernisasyon ng wikang pambansa?

A. 1987 B. 1974

322. Ang katumbas ng igkas ng titik Q sa kasalukuyan ay

A. Kuyu B. Kyu

323. Hanapin mo ako sa ng mga sasakyan.

A. Abangan B. Pag-abangan
324. Gagawin mo lang yan mo ako mahal.

A. Kung di B. Kungdi

325. ka na sa ilog. Nanginginig ka na.

A. Humango B. Umahon

326. Kinakailangang ang damit sa Huwebes.

A. Magyari B. Mayari

327. Limang oras na pero hindi ko pa rin ang nawawalang aklat.

A. Makita B. Mahanap

328. Pahalagahan ang pangaral ______________ hindi malihis ng lands. Anong


pangatnig ang angkop sa pahayag na ito?

A. Nang B. Habang

329. Nakapandidiri ang asong kalye na .

A. Madumi B. Dumumi

330. Anong salita ang nagsisilbing salitang ugat ng PINAGLABANAN?

A. Laban B. Labanan
331. Alin sa mga sumusunod ang pangungusap na may paksa?

A. Nagbabasa sila sa aklatan.


B. Kay ganda ng paglubog ng araw.

332. Huwag makisama kay Zoilo dahil buwaya siya. Ang salitang buwaya ay tumutukoy sa
kahulugang

A. Denotasyon B. Konotasyon

333. Uri ng pangatnig na ginagamit sa pagpili, pagbubukod at pagtatangi.

A. Pamukod B. Paninsay

334. Bantas na ginagamit sa pagitan ng panlaping ika at tambilang

A. Gitling B. Panaklong

335. Ano ang tawag sa tatlong magkakasunod na tuldok na ginagamit upang ipabatid na may
bahaging hindi sinipi mula sa talata?
A. Ellipsis B. Sintesis

336. Uri ng pagbabagong morpoponemiko na gumagamit ng pagpapapalit ng posisyon ng


ponema sa salita.

A. Asimilasyon B. Metatesis

337. Uri ng pagsusulat na ang pokus ay ang imahinasyon ng manunulat upang pukawin ang
damdamin.

A. Jornalistik B. Malikhain

338. Alin sa mga sumusunod ang nararapat sa komunikasyon na pasulat?

A. Lakas ng boses
B. Maayos na pagpapalugit

339. Ano ang kalabang mortal ng pakiking?

A. Talinghaga B. Ingay
340. Pahayag na pasaklaw na nabubuo sa pamamagitan ng personal na panlasa o pagpili.

A. Subjective generalization
B. Analytical generalization

341. Dulog pampanitikan na nagbibigay diing sariling panlasag bumabasa. Kilala rin ito bilang
reader-response theory.

A. Antropolohiya
B. Impresyonista

342. Proseso ng paghahatid ng saloobin, opinion, karunungan sa pamamagitan ng


makabuluhang tunog.

A. Pakikinig B. Pagsasalita

343. Nakikipagaway ka sa speaker. Ito ay pakikinig na

A. Pasibo B. Kombatib

344. Paraan ng pagbasa na ginagamit kung ang akda ay mahirap unawain.

A. Muling basa B. Masusis

345. Kung ang pangungusap ay, “Bilang at sukat kung mangusap ang dalaga”, paano
mailalarawan ang dalaga?

A. Mahirap unawain
B. Maingat

346. Isang uri ng pamamatnubay kung saan ang reporter ay lumilihis sa pamatnubay;
lumilikha ito ng sariling paraan sa mga gawaing pag- ulat.

A. Kombensyunal
B. Di-kombensyunal

347. Ibigay ang angkop na damdaming Ito ang dinadaanan upang maipaabot ng tagapagsalita
ang kanyang mensahe

A. Tsanel B. Fidbak
348. Ang pag-aaral ay nakatuon sa motibasyon ng mga mag-aaral sa paggamit ng
kompyuter sa paggawa ng mga sulatin. Natuklasan sa pag-aaral na ito na ang ganitong
pahayag ay ilalagay sa .

A. Abstrak B. Paradigm

349. Kung ikaw ay nagnanais lamang magpalipas ng oras, anong klaseng pagbasa ang
nababagay sa iyo?

A. Scanning B. Kaswal

350. Ang kahulugan ng: My bank account is in the read.

A. Bale-wala B. Malapit na maubos

351. Ano ang damdaming napapaloob sa “Bakit gabi na’y ‘di pa siay dumarating?”

A. Pagkatakot B. Pagkagalit

352. Sa taas ng mga bilihin ngayon kahit kahig ka nang kahig ay wala pa ring maipon. Ano ang
ibig sabihin?

A. Trabaho ng trabaho
B. Hanap nang hanap

353. Ang pahayagan ay hindi dapat maglathala ng anumang uri ng pagbibintang na


makasisira sa reputasyon nang di muna nagbibigay ng pagkakataon sa nasasakdal na
marinig ang kanyang panig. Ito ay
.

A. Patas na pamamahayag
B. Makatarungang pakikitungo

354. “You can count on me,” Ang pinakamalapit na salin nito ay .

A. Maaasahan mo ako
B. Bilangin mo ako
355. “Malalim ang bulsa,” ng kanyang Nanay. Ang ibig sabihin nito ay
.

A. Walang pera B. Kuripot


356. Siya ay may kutsarang pilak ng ipinanganak. Ito ay nangangahulugang
.

A. Masalita B. Mayaman

357. “Panahon na upang magdilat ng mata at makisangkot sa mga usapin.” Ito’y


nagpapahiwatig na .

A. Magising sa katotohanan
B. Idilat ang mga mata

358. “Ang lumalakad ng mabilis, kung matinik ay malalim.” Ano ang ibig sabihin ng
paalaalang ito?

A. Ang naglalakad nang mabilis ay hindi nakakapag-isip.


B. Ang nagmamadali ay madalas magkamali.

359. Mahusay “maglubid ng buhangin” ang taong gipit. Ano ang ibig sabihin?

A. Magsinungaling
B. Magpaikot-ikot

360. “Mahal kita, mahal kita hindi ‘to bola/ Ngumit ka man lang sana ako’t nasa langit na.” Ang
salitang pampanitikan na ginamit sa linya ng kanta ay nangangahulugang .

A. Sinisinta B. Napakasaya

361. Anong uri ng tayutay ang pahayag na ito? “Ang kabutihan mo sa buhay ang magiging
hakbang sa pag-unlad.”

A. Pagtutulad B. Pagwawangis

362. Alin dito ang pinakawastong salin sa pangungusap na: “I am sure he didn’t say that.”

A. Natitiyak kong hindi niya sinabi iyon.


B. Ako ay tiayk na hindi niya sinabi iyon.

363. Ang salin sa blangko para sa blank ay halimbawa ng pagtutumbas na


.

A. Panghiram sa Espanyol at pagbaybay sa Filipino


B. Paggamit ng leksikong Filipino 43
364. “Pinutol mo/ dagkung kahoy/ dahil ditto/ gumulong ang mga bato/ ania na, ania na, ania
na…” Ang linya ng awit ay nagpapatunay ng .
A. Diglossic na kkalagayan ng wika
B. Paggamit ng lalawigang antas

365. “Heto na, heto na, heto na, /wahh! Doo bidoo bidoo, bidoo, bidoo.” Alin ang nabuong
salita o tunog sa linya nabuo ayon sa teoryang pooh- pooh?

A. Wahh B. Doo bidoo

366. “Ang babae ay huwag mong tingnang isang bagay na libangan lamang kundi isang
katuwang at karamay sa mga kahirapan nitong kabuhayan.” (E. Jacinto) Ang tungkulin ng
wika sa linya ay .

A. Panregulatori
B. Pang-instrumental

367. Pen pen de sarapen/ de cuchillo de almasen/ haw haw de carabao batuten.
Pinatutunayan ng linya ang paglaganap ng wika dahil sa
.

A. Pananakop B. Heograpiya

368. Pinag-aaralang mabuti ng bawa’t abogado ang ng 1987.

A. Saligang batas
B. Batas pambansa

369. Ang paniniwala sa Diyos ay susi upang matamo natin ang .

A. Kaligtasan B. Salvasyon

370. Tapos na ang klase kapag tumunog na ang .

A. Batingting B. Kampana

371. Upang hindi madala ng agos ang barko, ibinaba ng mga mandaragat ang .

A. Timpulan B. Angkora
372. Ang aklat ni Dr. Tinsley Harrison ang sa pag-aral ng medisina.

A. Sandigan B. Pundasyon

373. Winasak ng Katrina ang maraming bahay sa New Orleans.

A. Bagyong B. Buhawi

374. Ang pagbibigay ng diploma ay nasa huling bahagi ng .

A. Programa B. Palatuntunan
375. Ang sanaysay na ito ay nagtataglay ng pangkaraniwang paksa at waring nakikipagusap
lamang.

A. Malaya B. Maanyo

376. Ang katangiang ito ng wika ay nangangahulugang ang wika ay nauunawaan ng


lahat at napagkasunduan ng isang lahi/pangkat.

A. Dinamiko B. Arbitraryo

377. Ito ay naguulat ng mga tunay na pangyayari batay sa pag-aaral, pananaliksik, o


pakikipanayam. Ngunit, ang paraan ng pagsulat ay kawili-wili. Ano ito?

A. Lathalain B. Editorial

378. “Kapapasok pa lang niya sa silid.” Ang pandiwa sa pangungusap ay nasa aspetong

A. Pangnagdaan B. Perpektibo

379. Paano mailalarawan ang isang akademing marunong tumanggap ng kritisismo para
sa ikagaganda ng pananaliksik?

A. Malikhain B. Bukas ang isipan

380. Ang wastong salin ng “You are the apple of my eye.”

A. Ikaw ay mahalaga sa akin


B. Katuwa-tuwa ka
381. Sa pangungusap na “Malakas ang boses mo”, ang salitang malakas ay isang

A. Panguri B. Pandiwa

382. Ang pagpapalitan ng mga ideya, opinyon, salaysay, sa pamamagitan ng mga sagisag
ay tinatawag na .

A. Talastasan B. Paglalahad

383. Saan kabilang na uri ng tayutay ang pahag na: “Kapalaran, huwag ka sanang mailap.”

A. Pagtawag B. Palit-tawag

384. Ang bahaging ito ng lingwistika ang siyang nag-aaral ng mga tuntunin kung paano
inaayos ang mga salita sa loob ng pangungusap

A. Syntax B. Pragmatika

385. Kung bibilangin ang pantig sa bawat taludtod ng tula, ito ang makukuha
A. Tugma B. Sukat

386. Ito ay uri ng pangungusap na tumutukoy sa pangyayaring pangkalikasan o


pangkapaligiran

A. Eksistensyal B. Phenomenal

387. Sa anong bahagi ng pananaliksik matatagpuan ang mga kaugnay na literature at pag-
aaral?

A. Kabanata I B. Kabanata II

388. Ang wastong kahulugan ng “The present problem is only a storm in a teacup” ay

A. May galit B. Bale-wala

389. Sa pangungusap na “Malakas ang boses mo”, ang salitang malakas ay isang

A. Pang-uri B. Panghalip 46
390. Alin sa mga sumusunod ang di-mahalagang salik sa pagtatalumpati?

A. Pagyayabang B. Okasyon

391. Ang simbolong kumakatawan sa mga bagay at mga pangungusap na nais ipahayag ng
tao sa kanyang kapwa ay

A. Wika B. Tunog

392. Tumutukoy ito sa mga salitang nakapagiisa at may kahulugan. Kilala rin itong
salitang-ugat.

A. Malayang morpema B. Di-malayang morpema

393. Anong anyo ng patanong na pangungusap ang: Dumaan ka na dito, di ba?

A. May karugtong B. Kabalikang anyo

394. Sa anong bahagi ng pananaliksik matatagpuan ang mga lugar at babasahing


mapagkukunan ng mga literature at pag-aaral.

A. Kabanata I B. Kabanata II

395. Science provides knowledge through disciplined observation. Which of the following is
not characteristic of scientific assertion?

A. Hearsay
B. Empirical support
396. Which of the following when perceived causes action on a reaction?

A. Stimulus B. Sound

397. Which among the following represents the smallest unit of life?

A. Embryo B. Cell

398. Which of the following statements about living things is false?

A. All living things have a nervous system


B. All living things are capable of reproduction
399. Which of the following best describes a group of cells that work together to perform a
function?

A. Organ B. Tissue

400. Which among the following are duplicated during the process of mitosis?

A. Chromosomes B. Centromeres

401. Plants create their own food by absorbing and processing sunlight. The ability to produce
one’s own food source is a metabolic process known as:

A. Autotrophy B. Homotrophy

402. Fungi absorb the nutrients from dead organisms. In an ecosystem what roles do they
play?

A. Carnivore B. Saprophytes

403. Which among the following cell organelles does not participate in cellular division?

A. Cytoplasm B. Ribosomes

404. Which of the following is an example of a symbiotic relationship?

A. Wolves working together to hunt in a pack


B. Weeds clinging to the roots of trees in your backyard

405. Which of the following is not an example of a naturally occurring sugar?

A. Glucose B. Mitose

406. Sugar compounds such as monosaccharides, disaccharides, and polysaccharides


are also classified

A. Carbohydrates B. Proteins
407. Polysaccharides, triglycerides, polypeptides, and nucleic acids are classified as
A. Acids and bases
B. Macromolecular

408. Which of the following are the components of fungal cell wall?

A. Cellulose, lipids, and nucleic acids


B. Chitin, proteins, and sugar

409. What organisms would most likely be in an artic environment?

A. Walrus B. Turtle

410. How are bats able to navigate in the dark without bumping into anything?

A. They emit sound waves whose returning vibrations can detect the presence of
objects
B. They have supersonic eyes

411. A species of fish lived in a lake. When a dam was constructed in the area, a group of fish
was separated and populated a new pond. They then developed differing characteristics
and became a distinct species. Which of the following concepts explains this speciation?

A. Geographical isolation
B. Behavioral isolation

412. This demonstrates the feeding connections between all life forms.

A. Nutrition cycle B. Food web

413. Plants capture energy from the sunlight by means of molecule known as

A. Adenosine triphosphate
B. Chlorophyll

414. Because it will adversely affect the populations’ food supplies, the most
disadvantageous source of energy is .
49
A. Geo-thermal B. Plants
415. Damage to DNA that is not repaired and then replicated can result in genetic disorders.
This demonstrates .

A. Mutation B. Pathology

416. Air, when mixed with sulfur oxide coming from motor vehicles and industrial plants falls
to earth giving effects to living things. This polluted air is known as

A. Acid rain B. Greenhouse rain


417. Air, flood, and water are essential elements in human existence. Carried by wind air
called _____ falls to earth poisoning fish and destroying vegetation.

A. Acid rain B. Greenhouse rain

418. You won the jackpot prize in the lottery. The prize money cannot be paid in currency byt
only in gold. Applying your knowledge in chemistry, select the largest amount of gold
from the choices below. (Au = 197).

A. 40.0 kilograms of gold


B. 22.4 moles of gold

419. The Kyoto protocol, which requires countries to reduce greenhouse emission, is a practice
that advocates .

A. Environmental protection
B. Ecological destruction

420. Which is true of metalloids?

A. Have properties of both metals and non-metals


B. Conduct heat and electricity less effectively than non-metals

421. An atom of silicon has a mass number of 28 and an atomic number of


14. How many protons are in this atom?

A. 28 B. 14
422. Element X has an electron configuration of 1S2 2S2 2P2. Based on information, element
X belongs to a/an family.

A. Coinage
B. Halogen

423. A toothpick can sit on the surface of water due to

A. Surface tension B. Buoyancy

424. When the North Pole is tilted towards the sun, it is summer and when the sun shines all the
time both day and night, what is this called?

A. Midnight sun
B. Northern light

425. What type of clouds produce thunderstorms

A. Cumulonimbus
B. Stratus

426. What is a pyroclastic flow?


A. Poisonous gas of a volcano
B. A mixture of volcanic gas, ash and other tephra

427. Rock fragments thrown into the air during a volcanic eruption are called:

A. Tephra
Lava B.

428. Which among the following properties is a measure of how easily a mineral can be
scratched?

A. Hardness
Luster B.

429. What is the numerical value for the hardest mineral on the Mohs hardness scale?

A. 10 B. 20
430. What part of the Earth is thought to be composed of iron and nickel?

A. Lithosphere B. Core

431. What is the partially molten layer of the Earth called?

A. Asthenosphere B. Lithosphere

432. If there are several earthquakes in a row, what is the name of the largest of these
earthquakes

A. Mainshock
B. Tertiary shock

433. What is the location on the Earth’s surface that is directly above the place where an
earthquake originates?

A. Epicenter B. Fault

434. Fats do not dissolve in water because fats .

A. Are nonpolar and water is polar


B. Are polar and water is nonpolar

435. Breaking up solid speeds dissolving in a liquid by .

A. Raising the temperature


B. Increasing surface area

436. The amount of substance having 6.02x1023 of any kind of chemical unit is called a(n):
A. Mass number B. Mole

437. According to the modern periodic law, the chemical properties of the elements are
periodic functions of their .

A. Atomic mass B. Atomic number

438. A horizontal row of elements on the Periodic Table is known as a(n):

A. Period or series
B. Group or family 52
439. If no more solute can be dissolved in a solvent at a given temperature, then that solution is
called:

A. Saturated B. Saturable

440. What intermolecular forces do geckos and other lizards use to stick to walls and ceilings?

A. Static ionic attractions


B. Van der Waals forces

441. Salts of which of the following metals are added to fireworks to make them brilliant red?

A. Copper B. Strontium

442. Alchemists in ancient Egypt tested gold for purity by remelting it and heating it. If the gold
became whiter, it contained:

A. Lead B. Silver

443. Which of the following is an example of a fungus?

A. Yeast B. Euglenoids

444. Which among the following elements can be drawn into thin wires and hammered into
thin sheets?

A. Cu B. Ag

445. A molecule or an ion is classified as a Lewis base if it has which of the following
properties?

A. Accepts a proton from water


B. Donates a proton to water

446. Which among the following has the most polar bond?

A. N2 B. HF
447. Which of the following best describes the amount of energy required to remove an
electron from a gaseous atom?

A. Ionization energy
B. Activation energy

448. What does a pH below 7 indicate?

A. Acidity B. Alkalinity

449. Which among the following gases is green in color?

A. Krypton B. Chlorine

450. The manager of textile factory considers it too expensive to treat liquid wastes so he
suggested digging a hole near the factory where the wastes can be stored. Is this
environmentally safe?

A. No, the liquid wastes will mix to the groundwater.


B. No, the liquid can be recycled.

451. Why are nights cooler when the sky is clear than when it is overcast?

A. Cooler air holds less vapour


B. There is no cloud cover to prevent heat from escaping

452. Water covers 75% of the Earth’s surface and yet some people still do not have enough
water to drink. Why?
1. Most of world’s water is salty
2. Most of our bodies of water are polluted by chemicals
3. Most freshwater are frozen in ice caps

A. 1 and 2 only B. 1, 2 and 3

453. Children born with a deficiency of thyroxin if deficiency is not checked.

A. Become mentally retarded


B. Have weak bones
454. The correct way of eating fruits is when eaten .

A. With an empty stomach


B. After meals

455. An eclipse of the sun throws the shadow of the .

A. Moon on the earth


B. Moon on the sun

456. The following are effects of global warming, except:


A. Changes in the aiming of seasonal patterns in ecosystem
B. Expansion of tropical diseases

457. John experienced a lack of growth hormone (GH) as a child. Now that John is an adult, it
is likely that he:

A. Has normal adult body proportions, but an extremely short


stature
B. Is of normal height, but has child-like body proportions

458. Which type of digestion occurs in the mouth when an individual chews a piece of bread?

A. Mechanical digestion only


B. Both mechanical and chemical digestion

459. One immediate cause of a decrease in the rate of photosynthesis is a reduction in the
availability of which of the following substance?

A. Oxygen B. Carbon dioxide

460. Vascular tissue that transports water in leaves connects directly to which of the
following?

A. Xylem in the stem


B. Root hairs in the epidermis
461. Which of the following statements is true about parasites and their relationships with
their hosts?

A. The parasite enjoys the benefits of a relationship in which


the host is harmed or killed
B. The host enjoys the benefits of a relationship in which the parasite is harmed
or killed

462. In computing data, a student computes up to the second decimal place. As a researcher
or scientist, what trait(s) do(es) the student possess?
1. Accuracy
2. Critical mindedness
3. Objectivity
4. Truthfulness

A. 1 and 4 B. 1 and 3

463. What is the main difference between prokaryotes and the eukaryotes?

A. The eukaryote has a nucleus, but the prokaryote does not


have.
B. The prokaryote has a nucleus, but the eukaryote does not have
464. Which of the following is a proper example of natural selection?

A. A community of lady bugs begins having offspring with darker


shells, in order to hide from their predators
B. Seagulls begin to eat so many fish so that only those with dark blue scale
color survive

465. Which of the following has the smallest mass when measured in an equal arm balance?

A. 1/8 sheet of pad paper


B. ½ sheet of pad paper

466. During periods of increased global temperatures, which of the following is most
likely to occur?

A. A decrease in atmospheric CO2


B. An increase in atmospheric CO2
467. Why is the Philippines much hotter than Canada? The Philippines:

A. Has days with more hours of light


B. Receives sun rays closer to vertical position

468. Geothermal energy, an energy resource in the country, is based on which


phenomenon?

A. There are more concentrations of heat in some places of


earth’s crust
B. Earth’s internal energy heats its surface more than the sun does

469. Which of these effects generally occurs as a result of warm air mass and a cooler air
mass converging at earth’s surface?

A. Winds lie down


B. Stormy weather patterns develop

470. Why do fisherfolks catch more fish during the new moon than during the full moon?

A. Fishes are attracted to the light from the fishing boat which
fisherman makes use of during the new moon
B. Fishes go near the surface during the new moon because the water is warmer

471. Bagoong smells because .

A. Air and volatile particles from bagoong mix and get spoiled further
B. Molecules from bagoong spread in air

472. Every morning when the sun shines through the glass jalousies of Maria’s window, she
sees a rainbow on her wall. Which could explain this?
A. The glass jalousies break up the beam of sunlight into the
colors of the rainbow
B. There are water droplets on the glass that act like a prism
473. Which of the following is true about light and seeing?

A. Light rays coming from an object meets lightcoming from a light source, and
then enters the eyes
B. Light hits an object, bounces and then enters the eyes

474. Why do leaves with green color look green in the sunlight?

A. They absorb the green light


B. They reflect green light

475. Why does the level of water in a beaker rise after a stone is placed in it?

A. The stone takes the place occupied by water


B. The stone has more mass than the water

476. Why do people feel their ears pop or crack when they are up in the air?

A. There are less air particles


B. There are more air particles

477. Which of the following activities help reduce water pollution?

A. Treating wastewater before it is discharged


B. Cleaning of soap instead of detergents

478. Of the following organisms, which are included under Kingdom Plantae?

A. Magnolidae B. Prokaryotes

479. Which of the following is the lowest rank of coal and is otherwise called brown coal?

A. Lignite B. Anthracite

480. Which element has Te as its symbol?

A. Tellurium B. Technetium
481. What is the term form slowing-down response by the frog’s circulatory system during
cold season?

A. Hibernation B. Estivation

482. Which of the following is a vitamin?

A. Niacin B. Pepsin
483. Which element has Ca as its symbol?

A. Carbon B. Calcium

484. Which of the following process involves chloroplast?

A. Conversion of light energy to chemical energy


B. Cell division

485. Aside from solid, liquid, and gas what is the fourth form of matter?

A. Plasma B. Colloid

486. Which of the following is a heterotroph?

A. Grasshopper B. Algae

487. Which type of reaction does the general equation (AB + energy A + B) show?

A. Endothermic synthesis
B. Endothermic decomposition

488. Iron is what type of magnetic material?

A. Ferromagnetic
B. Paramagnetic

489. Humans reproduce when an ovum is combined with a sperm leading to the development
of an embryo. This form of reproduction is called

A. Budding B. Fertilization
490. Of the following which is an example of physiological adaptation that allows survival
among plants or animals?

A. Frog’s powerful hind to jump away from predators


B. Walking stick’s unique shape and color

491. Which of the following activities help reduce water pollution?

A. Use of soap and detergents


B. Treating waste water prior to discharge

492. What is the main organ for respiration?

A. Lungs B. Nose

493. Dengue is an infectious disease transmitted by an organism classified under which


phylum?
A. Arthropoda B. Echinodermata

494. The source of water pollution is when water from a river is mixed with the contents of
a septic tank, this is called

A. Water condensation
B. Hypertrophication

495. The union of an ovum and sperm inside a test tube is

A. In vitro fertilization B. Pollination

496. What are the small hair-like structures that serve as a sensory function in eukaryotic
cells?

A. Cilia B. Actin filaments

497. What thin structures are essential for cytokinesis, amoeboid movement, and
changes in cell shape?

A. Pseudofilaments
B. Microfilaments
498. Which of the following process best explains the evolution of life forms or gradual
transformation of species?

A. Natural selection
B. Genetic drift

499. Which technology can lead to the formation of a new genetic trait?

A. Embryo transplant
B. Exposure to radiation

500. A rectangular book of steel has dimensions of 5 meters x 10 meters x 15 meters and
weights, 1, 000 N. How should this block be placed on a surface to exert the least pressure
on the surface?

A. On the 10 meters by 15 meters side


B. On the 5 meters by 15 meters side

GENERAL EDUCATION ANSWER


KEYS
1 A 31 B 61 A 91 B 121 B
2 A 32 B 62 B 92 B 122 A
3 A 33 B 63 A 93 B 123 B
4 B 34 A 64 B 94 A 124 A
5 A 35 A 65 B 95 B 125 A
6 A 36 A 66 A 96 A 126 A
7 B 37 A 67 A 97 A 127 A
8 A 38 A 68 A 98 A 128 A
9 B 39 A 69 A 99 B 129 A
10 A 40 B 70 A 100 B 130 A
11 A 41 B 71 A 101 A 131 A
12 B 42 A 72 B 102 A 132 B
13 B 43 A 73 A 103 B 133 B
14 A 44 A 74 A 104 B 134 A
15 A 45 B 75 A 105 A 135 A
16 A 46 B 76 A 106 A 136 A
17 A 47 B 77 B 107 A 137 B
18 B 48 A 78 B 108 A 138 B
19 B 49 B 79 B 109 A 139 A
20 A 50 A 80 B 110 A 140 A
21 B 51 B 81 B 111 A 141 B
22 B 52 B 82 A 112 A 142 A
23 B 53 A 83 A 113 B 143 A
24 A 54 A 84 A 114 B 144 B
25 A 55 A 85 A 115 A 145 A
26 A 56 B 86 A 116 B 146 B
27 B 57 A 87 A 117 A 147 A
28 B 58 A 88 B 118 A 148 B
29 A 59 B 89 B 119 B 149 B
30 B 60 A 90 B 120 A 150 A
151 A 191 A 231 A 271 B 311 A
152 A 192 B 232 B 272 A 312 A
153 B 193 B 233 A 273 A 313 B
154 B 194 B 234 A 274 A 314 A
155 A 195 A 235 B 275 A 315 B
156 B 196 A 236 A 276 B 316 A
157 A 197 A 237 A 277 B 317 A
158 B 198 B 238 A 278 A 318 A
159 B 199 B 239 A 279 A 319 A
160 A 200 B 240 A 280 A 320 A
161 A 201 B 241 B 281 A 321 A
162 A 202 B 242 A 282 B 322 B
163 B 203 A 243 B 283 A 323 A
164 A 204 A 244 B 284 B 324 A
165 B 205 B 245 A 285 B 325 B
166 A 206 B 246 B 286 A 326 B
167 B 207 A 247 B 287 B 327 A
168 A 208 B 248 A 288 A 328 A
169 B 209 A 249 B 289 A 329 A
170 B 210 A 250 B 290 A 330 A
171 A 211 B 251 B 291 A 331 A
172 B 212 B 252 B 292 A 332 B
173 A 213 B 253 A 293 A 333 A
174 A 214 B 254 B 294 B 334 A
175 B 215 A 255 B 295 A 335 A
176 A 216 A 256 B 296 A 336 B
177 B 217 B 257 A 297 A 337 B
178 B 218 B 258 B 298 A 338 B
179 A 219 A 259 B 299 A 339 B
180 B 220 A 260 B 300 A 340 A
181 B 221 B 261 A 301 A 341 B
182 A 222 B 262 B 302 B 342 B
183 B 223 A 263 B 303 A 343 B
184 A 224 B 264 A 304 A 344 A
185 B 225 B 265 A 305 B 345 B
186 B 226 A 266 B 306 A 346 B
187 A 227 B 267 B 307 A 347 A
188 A 228 B 268 A 308 A 348 A
189 A 229 B 269 A 309 A 349 B
190 B 230 A 270 B 310 B 350 B
351 A 391 A 431 A 471 B
352 A 392 A 432 A 472 A
353 A 393 A 433 A 473 B
354 A 394 A 434 A 474 B
355 B 395 A 435 B 475 A
356 B 396 A 436 B 476 A
357 A 397 B 437 B 477 A
358 B 398 A 438 A 478 A
359 A 399 B 439 A 479 A
360 B 400 A 440 B 480 A
361 B 401 A 441 B 481 A
362 A 402 B 442 B 482 A
363 A 403 B 443 A 483 B
364 B 404 B 444 A 484 A
365 A 405 B 445 A 485 A
366 A 406 A 446 B 486 A
367 B 407 B 447 A 487 B
368 A 408 B 448 A 488 A
369 A 409 A 449 B 489 B
370 B 410 A 450 A 490 A
371 A 411 A 451 B 491 B
372 B 412 B 452 B 492 A
373 A 413 B 453 A 493 A
374 A 414 B 454 A 494 B
375 A 415 A 455 A 495 A
376 B 416 A 456 B 496 A
377 A 417 A 457 A 497 B
378 B 418 A 458 B 498 A
379 B 419 A 459 B 499 B
380 B 420 A 460 A 500 A
381 A 421 B 461 A
382 A 422 A 462 A
383 A 423 A 463 B
384 A 424 A 464 A
385 B 425 A 465 A
386 B 426 B 466 B
387 B 427 B 467 B
388 B 428 B 468 A
389 A 429 A 469 B
390 A 430 B 470 A
PROFESSIONAL EDUCATION

501. Which of the following descriptions of a teacher is included in the preamble of the
Code of Ethics for Professional Teachers?

A. Persons of dignity and reputation


B. With satisfactory teaching performance

502. The effects of maternal employment on children’s achievement were found by a


number of researchers as .

A. Fully established
B. Hardly established

503. Why is babyhood often referred to as a “critical period” in the development of


personality?

A. At this time the baby is exposed to many hazards both physical and a
psychological
B. At this time the foundations are laid upon which the adult
personality structure is built

504. When is complete coordination of motor activities attained as established by


researches?

A. Childhood stage
B. Adolescence stage

505. Which of these theories holds that human activity is based on the interaction of stimuli
and responses?

A. Phenomenology
B. Association

506. What is the process by which an organism inherits the characteristics/ traits of the
parents?

A. Variation
B. Heredity
507. Kristle is competing with her mother for her father’s attention. She is therefore said to be
experiencing .

A. Electra complex B. Oedipus complex

508. To whom is classical conditioning theory always attributed for his experiment involving
the dog’s salivation as a reaction to the sound of the buzzer

A. Lewin B. Pavlov

509. How are institutions of learning encouraged to set higher standards of quality over and
above the minimum required for state recognition as provided for the Education Act of
1982?

A. Continuing professional education (CPE)


B. Voluntary accreditation

510. The main function of a philosophy of education is to:

A. Aid the learner to build his own personal philosophy


B. Define the goals and set the direction for which education is
to strive

511. The control and the administration of all educational institutions shall be vested in the
citizens of the Philippines as stipulated in
.

A. 1987 Constitution B. P.D. no. 176

512. The present military training in our school curriculum is an influence of

A. Sparta B. Athens

513. Which of these philosophies stresses the development of an individual capable of


reflective thinking especially that of being able to solve the problem he faces individually
or collectively?

A. Developmentalism
B. Experimentalism
514. Which of the following contributed to the establishment of secondary schools which is an
inclusion to basic education ?

A. Reformation
B. Realistic

515. The present Philippine Teachers Professionalization Act has its beginning on what
period of Roman History?

A. A.D. 100-AD 175-government increased its subsidy for


education
B. A.D. 274-259- government established a monopoly on education

516. Whose philosophy influenced the present emphasis on Character Education and
Values Education in our school system?

A. Gandhi B. Confucius

517. The singing of national anthem is an offshoot of the philosophical ideals of

A. Nationalism B. Socialism

518. Ejay, an adolescent combines his ability to use deductive and inductive reasoning in
realistic rules that he can respect and live by. When he does this, how does Ejay
perceives his environment?

A. He sees events apart from himself and other people


B. He sees the world through the eyes of the people

519. Manual aesthetic activities involving attitudes and feelings are primarily expressive
of emotions and values not thought. An example of this motor skill is .

A. Baking a cake with background music


B. Dancing and playing musical instruments

520. Parents and teachers are considered as authorities and models by children at the early
childhood stage. What does this statement imply?

A. Teachers and parents should serve as role models at all times


B. Parent-teacher conference should always be an activity in school
521. Which of these statements regarding professional teachers is the major difference in the
professionalization of teachers and teaching as promulgated in Presidential Decree
1006 and in Republic Act 7836?

A. Holder of valid professional license and certificate of


registration
B. Appointed on full time basis and on a permanent status

522. Which of these activities would be an effective way of avoiding or minimizing social
stratification in the classroom?

A. Assign leadership roles to the children of the upper social class


B. Avail the infuence of mass media for children of all classes

523. The constitution which provided the present philosophy of education was the contribution
of the Aquino Administration. Which of these statements is NOT consistent with our
educational philosophy?

A. Foster nationalism and patriotism


B. Appreciate the roles of foreigners on the historical saga of
the country

524. If children are cooperatively engaged with the teacher in a group project, the children
will discipline themselves as each member of the group exercises .

A. Obedience to the teacher


B. Moral compulsion

525. Operation “return to the basics” was launched by the Department of Education, Culture
and Sports not only to upgrade pupil achievement but also to .

A. Emphasize the importance of the 3Rs


B. Serve as the basis for a learning continuum

526. Which of the following embodies the operation “Return to the Basics”?

A. New Elementary School Curriculum


B. National Secondary Achievement Test
527. In a classroom it is possible to see the teacher doing the following to facilitate learning :
I. The class reads a workbook on the characteristics of animals
II. The class copies the characteristics of animals from books
III. The class goes out to the zoo to observe the animals
IV. The teacher shows posters of animals

Which of these teacher’s activities reflects an interactive environment?

A. I and IV B. III only

528. Which is NOT TRUE in the development of understanding in early childhood?

A. Ability to reason and to see relationship


B. Ability to explore their environment

529. Whose research established the first kindergarten also known as “a garden where
children can grow” ?

A. Froebel B. Herbart
530. Values development is integrated in all subjects in the NSEC while Values Education is
.

A. Offered as a separate subject


B. Emphasized in Science and technology

531. Which of the following is a mark of a good teacher?

A. Has the mastery of the lesson


B. Has the capability to implement corporal punishment

532. Under the learning to do, which of the following instruments must be acquired by Justin
so he can perform his work effectively?

A. Competence B. Insights
533. What do you think would be the action of a teacher who found out and has proven that
his principal is involved in the malversation of funds of their school?

A. Ignore what the teacher has discovered about this action of the principal
B. Present the charge to a competent authority

534. Teacher Abe is a neophyte teacher. One time a mother of one of his students confronted
and maligned him in front of his colleagues. How should Teacher Abe react on this kind of
situation?

A. Allow the mother to keep on maligning her until it’s turn to do the same
B. Wait until the emotion of the mother
subsides and invite her to discuss the concern with the
principal or guidance counselor

535. Ms. Gasat is always guided by the principle that she has a foremost responsibility as a
teacher. Given the following, which do you think is the main responsibility of Ms.Gasat?

A. Inspires student to interesting lessons


B. Guides students in the learning process

536. During the class reunion, Teacher Nancy learned that most of her classmates are
successful in their fields. Also, she found out that most of them are wealthy because they
have chosen a lucrative profession. Confronted with this situation, how should Teacher
Nancy react?

A. Leave the event so as to avoid being asked about her profession


B. Tell with pride that she is a teacher by profession

537. Roan as a principal is very much interested in a quality professional development program
of her teachers. Which of the following should she consider to realize this?

A. Responsive to identified teachers needs


B. Prescribed by top educational teachers
538. Ms. Sarah wants to help in ending Ms. Rose’s act of immorality but doesn’t have the
courage to confront her. What she did was to write and secretly distribute copies of
anonymous letter to her colleagues. What should have been done instead?

A. Talk to the married man with whom Ms. Rose is having an illicit affair
B. If the charge is valid, present such charge under oath before
her school head

539. The principal asks his good teachers to write modular lessons in Science, then he had
them published with his name printed as author. Which is unethical in this case?

A. He got the merit which was due for his teacher writers
B. He burdened the teachers with work not related to teaching

540. Teacher Ruth, a teacher for thirty two years, refuses to attend seminars. She claims
that her thirty two years of teaching is more than all the seminars she is asked to attend.
Are her actuation and thinking in accordance with the Code of Ethics for Professional
Teachers?

A. Yes, because she taught for thirty two years and may have mastered the
subject
B. No, a professional teacher, regardless of teaching experience,
ought to go through CPE

541. Operation “return to the basics” was launched by the Department of Education, Culture
and Sports not only to upgrade pupil achievement but also to .

A. Emphasize the importance of the 3Rs


B. Serve as the basis for a learning continuum

542. Which of the following embodies the operation “Return to the Basics”?

A. New Elementary School Curriculum


B. National Secondary Achievement Test
543. Education is a continuous process of experiencing and revisiting or reorganizing
experiences according to a progressivist. What does it mean?

A. Education takes place anytime and anywhere


B. Education goes on throughout life

544. The main contribution of the Arroyo administration in education is Republic Act no. 9155.
This provision .

A. Renamed the DECS to DepED


B. Established the study now-pay later system

545. The main purpose of compulsory education of the Constitution is to:

A. Develop students into responsible thinking citizen


B. Acquaint students with the historical development of constitution
546. Which of the following situations violates the principle of respect?

A. Teacher Dave tells his students that what Teacher Neil taught is essential.
B. Teacher Dang is giving special favor to students to please so that she can get a
remarkable result in the evaluation

547. If Teacher Lando states that specialization is knowing more and more about less and
less, hence it is better to be generalist. What kind of philosophy does he uphold?

A. Essentialism
B. Progressivism

548. Teacher Iah is an inspiration to almost all of the students. Her efficiency and effectiveness
in the profession is truly outstanding. Which of the following describes this attitude toward
her?

A. Progressivism
B. Idealism
549. Teacher Janus finds teaching in a multi-cultural classes very challenging. Which
among the following choices will alleviate the difficulty of addressing theses
challenges?

A. He must nurture diversity rather than practicing domination


and oppression
B. He must welcome one sided view rather than the recognition of
biases

550. Ms. Algie is the most admired pre-school teacher in her school. Which among the
following can best explain her being a good teacher?

A. She manages to instill control to her students


B. She adheres to the want of the parents for their children

551. Teacher Bong brought a salamander in the class during a lecture about amphibians. The
salamander is a device commonly known as REALIA. Teacher can bring realia ONLY
when .

A. Available
B. Feasible

552. Students of Teacher Jigs described him as someone who knows what he is talking about.
Teacher Jigs therefore exhibits a power known as
.

A. Expert power
B. Referent power

553. To show disapproval to the misbehavior of the student, Teacher Prince clears his throat
and looks intently at the erring student. This classroom management style is commonly
known as .

A. Proximity control
B. Signal interference

554. When choosing an instructional aid or device, the primary consideration of the
teacher would be .

A. Suitability B. Efficiency
555. Which of the computer-based instructional tools can help you revise written work such as
short stories and essays?

A. Desktop publishing
B. Word processing

556. Ms. Bhum wants to show to her class a magnified picture of the Mt. Pinatubo’s crater fixed
on a bond paper. Which of the following tools can she use?

A. Overhead projector
B. Opaque projector

557. As teacher employs the project based multimedia learning (PBML) strategy, what are
some limitations teachers expect from the encounter?

I. There is a need for extending the time to use several media


II. The presentation of the product is not an easy task
III. The technology skills to produce a product may be lacking

A. I, II, III B. II and III only

558. B.F. Skinner is a known psychologist and the one who first described operant conditioning.
Which of the techniques is an application of operant conditioning?

A. Project method
B. Computer-assisted instruction

559. Which of the following choices is considered as social force that affects the school and
the curriculum?

A. Changes in gender roles


B. Learner’s characteristics

560. Which of the following is an expression of child’s interest in his body?

A. Looking at themselves in the mirror


B. Looking the picture of an adult men and women
561. Motor development is manifested by a particular child who ?
A. Learns how to walk, run, and jump
B. Recognizes the different sizes of toys given to him

562. Which among the following drugs is commonly used for children with ADHD?

A. Ritalin B. Thorazine

563. What is the main reason why children with ADHD have limited learning skills?

A. Are given sedatives which make them listen


B. Act on impulse and cannot concentrate

564. Many concerned parents commonly make the mistake of .

A. Unintentionally rewarding their children for creating stress


B. Unintentionally creating high levels of stress for their child

565. Stuttering is commonly caused by .

A. Problem with a physical origin


B. Side effect of authoritarian parenting

566. How can parents foster initiative and independence in children?

A. Mastering psychomotor skills


B. Encouragement from parents when a child plans and carries
out a task

567. Which stage considers the importance of teachers, peers and adults outside the home in
shaping attitude toward oneself?

A. Integrity versus despair


B. Industry versus inferiority

568. When a child manifest autism, self-destructive behavior and echolalia, the child might be
showing the symptoms of ?

A. Anorexia nervosa
B. Childhood autism 75
569. When Kevin’s moral choices are determined by the direct consequences of
actions, he is most likely in the stage of ?

A. Post conventional B. Pre-conventional

570. What is the motivation of Lester who paints for the sheer enjoyment of creating artwork?

A. Intrinsic B. Extrinsic
571. Providing variety of learning activities to students is a characteristic of a teacher who
understands the principle of .

A. Facilitating learning with emphasis on individual differences


B. Allowing the student to be exposed to various teaching
techniques

572. Students who are disobedient and who display negative attitudes towards others are
best handled by a teacher who will .

A. Take every opportunity to praise him for every positive


attitude displays
B. Detain him after office hours for him to do what he has been ordered

573. Which of the following develops critical thinking skill among the students?

A. A willingness to suspend judgment


B. Blind obedience to authority

574. A child who always fights with his /her classmates, who has a very short attention span,
and who has frequent tantrums is believed to be sufferring from

A. Mental retardation
B. Attention deficit hyperactivity disorder

575. Teacher Selwyn has been lecturing for more than an hour and he noticed that students
are not anymore able to absorb additional information. This phenomenon is known as
.

A. Stagnation B. Plateau of learning 76


576. Planned ignoring, signal interference and proximity control are techniques used in
.

A. Life space interviewing


B. Managing surface behavior

577. A foreigner who is studying here in the Philippines was turned off by the Filipinos way of
eating balut and frogs. This feeling is called
.

A. Culture phobia
B. Culture shock

578. Who among the following claimed that children are natural learners and therefore must
be taught in natural settings?

A. Montessori B. Froebel

579. The concept that learning to read or write does not happen quickly but is build upon many
small steps that occur over the course of the child’s early childhood.

A. Emergent literacy
B. Innate literacy

580. A boy is closer to his mother and a girl is closed to her father. These instances are under
.

A. Oepidal complex B. Phallic stage

581. Laughing at a two year old child who uttered a bad word is not a proper thing to do because
in this stages of the child’s life, the child is
.

A. Considering the views of others


B. Distinguishing right from wrong

582. The school director emphasizes the necessity of clean and green environment that
contribute to effective teaching and learning. This is an example of .

A. Providing an atmosphere conducive to learning


B. Establishing rapport between teachers and pupils 77
583. The teachers are facilitators of learning. Which among the following negates this
principle?

A. Uses pro-active discipline methods


B. Focus on the background of the students

584. The nearest to the real thing according to Edgar Dale’s Cone of Experience is .

A. Watching demo
B. Attending exhibit

585. This is the pre-planned collection of sample of student works, assessed results and other
output produced by the students .

A. Portfolio
B. Observation report

586. Which of the following statements is one of the strengths of an autobiography as a


technique for personality appraisal?

A. It can replace data obtain from other data techniques


B. It makes possible presentation of intimate experience

587. Student Raj was asked to report to the Guidance Office. Student Raj and his classmates at
once remark, “What’s wrong?” What does this mean?

A. Reporting to a guidance office is often associated with


misbehavior
B. Guidance counselors are perceived to be “almighty and omniscient.”

588. The discriminating index of item number 15 is 0.45 means that

A. More students from the upper group got the item correctly
B. More students from the lower group got the item correctly

589. What is the main advantage of using the table of specifications when constructing periodic
test?

A. It increases the validity of the test result


B. It improves the sampling of content areas 78
590. Which of the following is an example of norm-referenced interpretation?

A. Jiah’s test score is higher than 89% of the class


B. Fritz solves five problems correctly out of thirty problems

591. Which of the following assessment techniques best assess the objective “plans and
designs an experiment to be performed.”

A. Rating scale B. Checklist

592. What type of measure of variation is easily affected by the extreme scores?

A. Quartile deviation B. Standard deviation

593. Standard deviation is to measure of variation as is to measure of central tendency

A. Mean B. Mode

594. Which statement/s is/are true in constructing matching type of test?


I. The options and descriptions are not necessarily homogenous
II. Description in Column A and option in Column B
III. The options must be greater than the description
IV. The directions must state the basis of matching

A. I, II and III B. II, III and IV

595. A type of error committed in grading the performance of the students by the rater who
avoids both extremes of the scale and tends to rate everyone as average

A. Logical error B. Central tendency error

596. Which of the following should be AVOIDED in constructing true or false test?
I. Verbal clues and specific determiner
II. Terms denoting definite degree or amount
III. Taking statements directly from the book
IV. Keep true and false statement the same in length

A. I and III only B. I, II and III 79


597. Which of the following tests can best effectively measure higher order of cognitive learning
objectives?

A. Objective test
B. Extended essay test

598. To increase the difficulty of a multiple choice test item, which of the following should be
done?

A. Make the options homogeneous


B. Make the options equal in length

599. Teacher Abegail wants to establish the reliability of his test in Biology. Which of the
following will he accomplish?
I. Administer a parallel test
II. Split the test
III. Construct a variety of items
IV. Administer the same test twice

A. I, III, IV B. I, II, IV

600. Teacher Tiffany constructed a matching type test item. In her column of descriptions are
combination of presidents, current issues, and sports. Which rule of constructing a
matching type test item was NOT followed?

A. The descriptions must be homogeneous


B. The descriptions must be heterogeneous

601. Which of the following does NOT belong to the group when we talk about projective
personality test?

A. Sentence Completion test


B. Word Association test

602. Which of the following is the main purpose of administering a pre test and post test to
the students?

A. Measure gains in learning


B. Measure the effectiveness of instruction
603. Which is the first step in planning a periodic test?

A. Construct a table of specification


B. Go back to the instructional objectives

604. Below is the list of methods used to establish the reliability of a test, which method is
questionable due to practice and familiarity?
A. Kuder Richardson
B. Test retest

605. What is the meaning of TOS in the parlance of test construction?

A. Table of Specifications
B. Table of Specific Test

606. Which is implied by a positively skewed score distribution?

A. The mode is high


B. Most of the scores are low

607. Which of the following is a result of compressing a file?

A. The file is deleted


B. The file size is smaller

608. Which of the following are the basic components of curriculum design?

A. Content, Assessment and delivery


B. Assessment, Teaching strategies and textbooks

609. The following are examples of Learner-Centered Design EXCEPT

A. Child-centered design
B. Humanistic-centered design

610. Which of the following statements about computer viruses is TRUE?

A. Files damaged by computer viruses can be cured


B. Compressed files can never be damaged by
viruses
611. Mrs. Grachie dreams to organize a seminar with a known poet from another country but
she cannot afford to spend for the transportation of the said guest. Which of the following
can she use so she won’t need to spend much?

A. Instant messaging B. Video conferencing

612. Teacher Nemiah wants her students to express their opinions regarding environmental
problems. Which of the following can she use to do this?

A. Forum B. Blog

613. Ms. Vanessa will be absent for two days because of a national conference. She
wants her students to work on a certain module. What is the fastest way of sending the
module to her students while she is away?
A. Share it to all her students in her live account
B. Send the module through group instant messaging

614. Teacher Grace is assigned to handle a multi-grade class. What instructional material
must be used to provide the needs of each grade?

A. Differentiated materials to cater to different levels


B. Multisensory materials

615. A document published by a school district that identifies rules of behavior that must be
followed by anyone using the school district computers, network or internet connection

A. Fair use B. Ethical internet use policy 616. Which of the

following are the rationales behind using technology in


the classrooms?
I. Motivational
II. Unique instructional capabilities
III. Increased Teachers’ productivity

A. II and II only B. I, II and III


617. Using bloom’s Taxonomy, the highest among the following is

A. Critical comprehension
B. Critical evaluation

618. Among the following educators, who proposed the placement of children in a
“prepared environment”?

A. Montessori B. Froebel

619. To improve comprehension and retention among the students, the teacher’s best option
would be to use

A. SQ3R B. Contextual Clues

620. Teacher Nerry would like to compare and contrast plant cell vs. animal cell. She would
most likely use:

A. Venn diagram B. Tree diagram

621. The students of Teacher Panyang feel that their teacher has an “eye behind her head” This
characteristics of the teacher is known as

A. Referent power B. Withitness

622. Teacher Prince asks his students what they already know about the topic before starting
a new topic in his subject. What concept of cognitive development is exhibited in the
situation above?

A. Assimilation B. Schemata

623. During the distribution of the report card, which of the following must be the foremost
concern of a teacher?

A. Discuss the projects of the school


B. Discuss the progress as well as the deficiencies of the student

624. How will you classify the purpose of the school as it concerns with the training and
preparation of citizens for the world of work?

A. Economic purpose
B. Social purpose
625. After listening to the homily of the Priest about fidelity, Julian has a moment of reflection.
His understanding of the value of fidelity has become deeper as he relate this to his past
experiences. This typifies what kind of philosophy ?

A. Constructivism B. Reconstructivism 626. Teacher Carl

is a new teacher. He realizes that handling his students’


misbehavior is a very demanding aspect of the classroom
management. In this regard he thought of giving up teaching. What advice can you give
him?

A. Report every student’s misbehavior to the principal


B. Set the ground rules for the whole class

627. Which among the following philosophers considered habits and reasons as equally
important forces to be cultivated in education?

A. Aristotle B. Jerome Bruner

628. Which of the following is the most important component of educational reform?

A. Implementing a better curriculum for students


B. Allowing the students to participate more

629. The failure in the test of independence among Filipino students can be attributed to

A. High degree of dependence to authority


B. Strong family ties

630. Teacher Justin is admired for being an effective classroom manager.


He is not only friendly but at the same time .

A. Business-like B. Analytical
631. Which of the following would be the most fitting action of a teacher who is having a
relationship with his/her student?

A. Defer the relationship until they are ready to admit it


B. Continue the relationship and exercise utmost professional
discretion about this 84
632. Some children go through a period of intense appetite when they eat or chew on all sorts
of inedible substances. This is called:

A. Pica B. Enuresis

633. Mrs. Grace Binay was not accepted by a certain company because of her age. This
discrimination based on age is called

A. Ageism B. Senilism

634. Jco’s parents do not want their child with ADHD to undergo drug treatment, their
better alternative would be

A. Behavior modification of behavior management


B. Punishment

635. Ripple effect can also be seen in misdemeanor. The teacher should therefore

A. Immediately respond to misbehavior


B. Be consistent in classroom management

636. The best example of Operant Conditioning among the following is


.

A. Using reinforcement
B. Fostering conducive learning environment

637. A child submitted a poorly written report but packaged with startingly colored paper cover.
This showcases

A. Subtance over “porma”


B. porma” over substance

638. Jeff has inherent skills in taking care of plants. It is highly possible that he has

A. Naturalistic intelligence
B. Intrapersonal intelligence
639. According to Erikson, a child who is cold towards that people around him might have failed
to attain what basic goal based on psychosocial development?

A. Trust B. Initiative
640. What is the most likely characteristic of children aged 3 to 5 according Erikson?

A. Mischievous B. Ego-centric 641. Metacogniton

is primarily characterized by .

A. Formulating hypothesis
B. Thinking about their thinking

642. A person who is friendly and has a capacity to make people laugh possesses .

A. Intrapersonal intelligence
B. Interpersonal intelligence

643. A student is finding it hard to read. When the guidance counselor traced the child’s history,
the counselor was able to find out the student came from a dysfunctional family, Aside from
that, the child was abused and neglected. What could have caused the student’s
reading disability?

A. Emotional factors
B. Poor teaching

644. As children move into the elementary school years, they direct their energy toward mastery
knowledge and skills. Which stage of Erikson’s theory is characterized here?

A. Industry VS. Inferiority


B. Identity VS. Role Confusion
645. To whom does the word teacher refer?
I. Full time teachers
II. Part time teachers
III. Guidance counselors
IV. Librarians
V. Division Superintendent

A. I,II, and V B. III and IV 646. Which is true of

LET passers?

A. Every LET passer shall be required to take the Professional


Oath before practicing as a Professional Teacher
B. Taking the Professional Oath before practicing as a Professional
Teacher is optional for LET passer

647. Are vocational teachers required of professional license?

A. Yes B. No

648. Can a failed examinee qualify for the position of para-teacher?


A. Yes, if his LET rating is below 75
B. Yes, if his LET rating is 70 and above 649. When

should teachers support one another?

A. At all times for whatever cause


B. When the best interest of learners is at stake ina controversy

650. What does the Teacher Education Development Program signify as a prerequisite for
employment of teachers in basic education schools?

A. Licensure examination for teachers


B. National standard competencies among teachers

651. How do orientation sessions help the new teacher?


I. Be attuned to the school environment
II. Learn guidelines to follow
III. Develop good relations

A. II and III B. I, II, III


652. Former DepEd Director Lolita Andrada described her humble early schooling: a silong
(ground floor of a house) converted into a classroom. Which best characterizes the
situation?

A. School-home connection B. Rural village school 653. What is the

overall stream for education, growth and fulfillment in


the teaching profession?

A. Merit and promotion B. Lifelong career

654. Among the following, which directly qualifies the teacher candidate to be registered at
par with other professions like medicine, law, nursing, and the like?

A. Licensure examination
B. Membership in professional association

655. In essence, what is the new Performance Appraisal System for Teachers (PAST)?

A. Competency-based B. Skill-based

656. Among active participation of school officials and teachers in the community, which of
the following is not appropriate due to prevailing religious sentiments?

A. Literacy assistance for out of school children/youths


B. Promoting contraceptives for planned parenthood

657. Among rights of the schools, which is not provided by the law?
A. Right for basic education to determine subjects of the study
B. Right for institutions of higher learning to determine academic
grounds for admission.

658. What is known as a self-appraisal for professional growth that is acceptable and useful
for recognizing weakness and strengths for a new beginning teacher?

A. Self-evaluation
B. Students’ evaluation
659. In order to assist new teacher, which is the most effective way to clarify the schools’
goals and responsibilities early in the first year?

A. Student’s handbook B. Orientation

660. Transparency international, a global corruption watchdog, rates the Philippines as one of
the most corrupt nation in Asia. What educational program is relevant to this situation?

A. Values education
B. Socio-economic development

661. The Transparency International’s perception that the Philippines suffer a cultural
malaise of corruption, what component of our character needs to be further
developed along the Learning To Be Pillar of education in the 21st century?

A. Intellectual-emotional component
B. Ethical-spiritual component

662. What is the cultural trait of conflicting values that aims to please people in different
venues and situations rather than abide by principles?

A. Kanya-kanya system
B. Split personality

663. What best describes “puwede na” mentality vs. excellence in service/ work?

A. Arduous preparation
B. Resignation to mediocrity

664. A teacher introduces herself as teacher only. What does this imply?

A. The teaching profession is the lowest paid profession


B. She takes no pride in the teaching profession

665. How is collaboration among teachers for sustained professional practice teachers
best achieved?

A. Mentoring system by experienced teachers


B. Interdependent work for cohesion
666. From the Constitution, what is the condition for allowing students to be taught religion in
public schools?

A. Outside of school hours only


B. Upon written permission by parents

667. Under R.A. 7722 or Higher Education Act of 1994, which agency has direct supervision
and management of teacher education institutions or colleges of education in the
country?

A. Commission on Higher Education


B. Teacher Education Council

668. Along the aim towards labor for integrity, which aspect of work needs to be taught to future
employers and employees in the 21st century workplace?

A. Industry skills and orientation


B. Loyalty for company and work

669. Which of the following is a quality of an engaging learning environment most reflective of
Education For All (EFA)?

A. Improvised to enhance learning for all types of


learners
B. Student-centered learning activities

670. Educated in a religious school, Sansa goes to confession every day to be free of any kind
of sin. How do you characterize Dona’s moral attitude?

A. Scrupulous B. Pharisaical

671. How should teachers regard new trends and reforms in education?

A. As a development need
B. As a wave in millennial change

672. How does the “humaneness” of the teacher best describe when he/ she is full of interest
and enthusiasm in the work of teaching?

A. Responsiveness
B. Perceptiveness
673. Of the following, which best describes peer teacher-and-learning?

A. Individual B. Collegial

674. To survive in the global community and in the twenty first century, one must be a:

A. Digital expert B. Digital immigrant


675. What observation tests to the fact that the student’s motivation vary according to socio-
cultural background?

A. Genetic endowments may show gifted endowments among the young


B. Children from low-income household meet more
obstacles in learning

676. Which indicator is most useful for assessing quality of schooling?

A. Cohort survival rate


B. Net enrollment rate

677. The world has become a global village. What sound practice is expected of
teachers?

A. Respect of diverse culture


B. Ethnic cleansing

678. Of the following, which is the most powerful source of values like beauty and
goodness outside the school system?

A. Nature
B. Mass media

679. Among components of a morally mature person, which demonstrates that a teacher
seeks peaceful resolution of conflict?

A. Working for peace/reconciliation


B. Accepting responsibility for choices
680. Which of the following refer to teamwork among teachers in classroom teaching?

A. Peer evaluation
B. Peer teaching

681. Of goals of education, which relates to the strengthening of our society’s sense of
belonging and identity?

A. Moral character
B. Citizenship

682. Inculcating moral maturity among students, which of the following relates to beliefs
and ideals?

A. Respecting freedom of conscience


B. Promoting human equality

683. Among core/basic life skill, which relate to recognizing and giving worth to ourselves
as persons?
A. Self-esteem
B. Empathy

684. What philosophical motive was behind public education during the American colonial
period?

A. Secularism
B. Utilitarianism

685. Students must be taught self-responsibility is the desire of the


teacher.

A. Existentialist B. Constructivist

686. Values are subjective. Your value may bot necessarily be my value.
What subscribe to this thought?
I. Progressivist
II. Pragmatists
III. Essentialist

A. I and III B. I and II


687. Which aspect of multi-intelligence is enhanced by asking students to work on a physical
model of the atom after a teacher’s discussion on the subject of the atom?

A. Interpersonal
B. Kinesthetical

688. John Dewey said, “An ounce of experience is better than a ton of theory.” To which does
this statement point?

A. The need for experience


B. The primacy of experience

689. In Piaget’s Concrete-Operational stage in cognitive development, which refers to


the ability to order or arrange things logically on dimension such as weight, volume,
or size?

A. Seriation
B. Conservation

690. In Piaget’s Formal Operation Stage in cognitive development, which of the following is not
among the characteristics of the child’s thinking ability?

A. Hypothesis making
B. Knowing properties (number, mass, volume, etc)

691. In Erikson’s Stage Theory of Development, which affirmation belongs to the stage of
Identity versus role confusion?
A. I get a good deal of pleasure from working
B. I wonder what sort of person I am

692. What is the degree of moral certitude of Vanessa who entered into marriage only out of
obedience to her parents, but uncertain whether she wanted marriage at all?

A. Probable B. Doubtful

693. Which is the most basic in Maslow’s hierarchy of needs?

A. Socialization B. Actualization
694. Among mistaken goals in the Acceptance Approach to discipline, what happens when
students seek to hurt others to make up being for being hurt or rejected?

A. Revenge seeking
B. Power seeking

695. Among mistaken goals in the Acceptance Approach to discipline, what happens when
students feel helpless and rejected so that they remove themselves rather than
confront the situation?

A. Withdrawal
B. Attention getting

696. How will you make a child memorize the table of multiplication from the point of view of
Pavlov?

A. Make them understand the meaning of multiplication


B. Call the child’s attention every time you recite the table for
them

697. In the 5’S Formula to create a conductive learning environment, which S stands
cleanliness of the work place?

A. Shine
B. Systematize

698. Among mistaken goals in the Acceptance Approach to discipline, what happens when
students are not getting the recognition they desire, continually seek to help, and refuse to
work unless the teacher hovers over them?

A. Withdrawal
B. Power seeking

699. What is being shown by teachers who ignore slow-learners and take pride in the work of
fast learners?

A. Hurling invectives
B. Discrimination
700. What provides the most direct type of learning, but is difficult to supply in the traditional
classroom?

A. Real-life experience
B. Multisensory instructional aids

701. Among thinking skills in learning, what includes fluent, flexible, original and elaborate
thinking?

A. Divergent thinking
B. Convergent thinking

702. Of the following effects on learning, what is the effect of assigning various sections of
the newspaper, and allowing choice depending on the learner’s choice?

A. Allowing different interests


B. Encouraging participation

703. Learning is influenced by social interaction and interpersonal relations.


What must a teacher do?

A. Make students feel good about themselves


B. Make students work collaboratively

704. How can new information be made more meaningful to students?

A. Relating it to knowledge they already know


B. Increasing retention of new knowledge

705. Which of the following best relate to basic literacy to learners?

A. Text-based learning
B. Field experience

706. What is the initial knowing process learned by the individual that revolves around
practical problem-solving?

A. Thinking
B. Listening
707. What condition afflicts child learners who have to count on fingers to tell whether 5 are
more than 4?

A. Neurological B. Mental

708. In learning complex matters, what is important to give focus on?

A. Separate and diverse thoughts and beliefs


B. Meaning from information, experience, and beliefs 709. Which is
not an aim in having partners in team learning?

A. Focused attention B. Cooperative work

710. Among the components in the instructional framework for learning strategies, which is
demonstrated by teacher Ana who assigns homework for students to do on their
own?

A. Independent practice B. Consolidation

711. Someone said, “Experience without theory is blind, but theory without experience is
mere intellectual play.” This means that:

A. Theory and experience must go hand-in-hand


B. Theory is more important than experience

712. Teacher A set 85% accuracy in a test on predicting the kind of weather given 5 different
atmospheric conditions. Mary obtain a score of 82% that can be interpreted as

A. She is 3% short of the set percentile score


B. She did not meet the set standard by 3%

713. Of the following, which is the best way parents can support the learning development
of their children?

A. Interest and assistance in class assignments


B. Checking report card grades

714. What characteristic differentiate spiritual intelligence or spiritual quotient as developed


by Harvard University, from sectarian religion (E.g. Christian, Buddhist, Jewish, etc.)?

A. Creedal values
B. Sectarian values 96
715. Sequence the following events on the historical development of reading:
I. Greek letters and the Roman alphabet were developed
II. Through the Semite’s ingenuity, sounds, and symbols gave rise
to the Phoenician alphabet
III. People used pictures and characters to convey messages
IV. Researchers showed the processes of reading, comprehension,
and interpretation

A. I, II, III, and IV B. III, II, I and IV

716. In Grace Godell’s Reading Skill Ladder, which step should come last, and only when
needed?

A. Using parts of a book


B. Using the dictionary

717. In Grace Godell’s Reading Skills Ladder, which is at the very bottom and needed to move
up to the sequential ladder of reading skills?
A. Phonetic analysis
B. Basic sight words

718. Among models of reading strategies, what did student Donald adopt when he reads back
and forth, attending to both what is in his mind and what’s on the page?

A. Bottoms-up
B. Interactive

719. What is known as learning a new word by studying its roots and affixes?

A. Morphology
B. Phonetic analysis

720. What is an example of a book or writing intended for moral knowledge of the very young
readers?

A. Jack and the Beanstalk


B. Les Miserable
97
721. Which of the following does not belong to the meta-cognitive reading activities?

A. Testing
B. Sampling

722. Among specialist in reading, who are mainly concerned about reading as a thinking
process that involves the recognition of printed or written symbols which serve as thought
stimuli?

A. Linguists
B. Psychologists

723. What is mainly addressed by early intervention program for children with disabilities, ages
0 to 3 years old?

A. Ensuring inclusion for special children


B. Early growth development lag

724. Who is best equipped to lead in the formulation of an individual educational plan for
special children?

A. Special education teachers


B. Parents of special children

725. To what problem type do students belong, who have short attention span, unable to
sustain concentration by being easily affected by sights, sounds or speech?
A. Withdrawn
B. Distractible

726. On categories of exceptionality in the young, what is difficulty in focusing and


maintaining attention, and/or recurrent hyperactive and impulsive behavior?

A. ADHD
B. Emotional/conduct disorders

727. How is the disorderly behavior of children classified when they tell lies?

A. Moral B. Social 98
728. In preparing classroom tests, which of the following checklists is the LAST among steps
in tests preparation?

A. How are the test results to be reported?


B. How are the test scores to be tabulated?

729. Tests should be non-threatening, but which of the following actually threatens students?

A. Surprise quiz
B. Scheduled exam

730. The test questions in Teacher Dae Dae’s test were confusing and subject to wrong
understanding, especially to poorer students. What was wrong with the test?

A. Unclear directions B. Ambiguity

731. In her tests, Teacher Tomden made tests that were either too difficult or too easy. What was
wrong with her tests?

A. Unclear directions
B. Inappropriate level of difficulty of the test
items

732. What is wrong with test items that give clues to answers to questions?

A. Tests brevity
B. Poor construction

733. With manner of answering as a criterion, which of the following belongs to supply
type of test?

A. Completion
B. Multiple choice

734. What primary response factor is considered by Essay questions?


A. Wide sampling of ideas
B. Factual information

99
735. In using Essay questions, what of the following are these tests susceptible to, such
as to make them less reliable measure broad knowledge of the subject matter?

A. Bluffing B. Time consuming

736. Among standardized tests, which reveals strengths and weaknesses for purposes of
placement and formulating an appropriate instructional program?

A. Achievement tests
B. Diagnostic tests

737. Among standardized tests, which is to measure cognitive abilities, and administered
either as a group or individual test?

A. Intelligence test
B. Achievement test

738. What kind of assessment helps teachers determine gaps in learning a subject matter?

A. Formative assessment
B. Summative assessment

739. An entering college would like to determine which course is best suited for him. Which
test is appropriate for this purpose?

A. Aptitude test B. Diagnostic test

740. Among recording devices during assessment, which can the student use to describe
behavior and context which observations may be forgotten or remembered
incorrectly?

A. Anecdotal record
B. Self-assessment

741. Among written categories of assessment methods, what did teacher Maggie Lagid use
when she assessed the stock knowledge of her students through questioning in an
open class?

A. Oral questioning
B. Performance test

100
742. Self-evaluation can be done in various ways, but this is not one of them:
A. Use of an evaluation instrument
B. Peer feedback session

743. Which of the following are alternative assessments?

A. Portfolio, exhibits, journals


B. Paper-and-pencil test, demonstration, reports

744. What is an alternative assessment tool that consists of a collection of work artifacts or in
progress accomplishment by a targeted clientele?

A. Rubric
B. Portfolio

745. Of the following, which is the most conducive for authentic assessment?

A. Performance drill
B. Real world application

746. Among written categories of assessment methods, what instruments did Mona use when
she used a check list in order to grade the dramatic performance of students in play?

A. Performance test
B. Product rating scale

747. In order for assessment to have clarity of learning targets, outcomes must be stated in
behavioral terms so that this can be measured. Which of the following shows this
desired clarity?

A. To practice reading with competence


B. To improve the speed of reading competence

748. In outcomes based education, which of the following are not considered
outcomes?

A. Products B. Procedures
749. Among written categories of assessment methods, what instruments did Science
teacher Julian use when he prepared a list of behavior that make up a certain type of
performance in the use of a microscope?

A. Product rating scale


B. Performance test

750. What is the criterion-referenced test designed to determine?

A. Performance level on a specific skill


B. Performance on a wide content of coverage
751. Of the following subjects, which does not belong to performance- based subjects in
which direct instruction is effectively used?

A. Mathematics B. Science

752. What does it mean if student Pete got a 60% percentile rank in class?

A. He scored better than 60% of the class


B. He scored less than 60% of the class

753. Which measure of central tendency is most reliable when scores are extremely high and
low?

A. Median B. Mean

754. What should be done with the test item whose difficulty index is .98?

A. Reject it B. Revise it 755. What does the test

mean if the difficulty index is 1?

A. Very difficult B. Very easy

756. What is the meaning of a negative correlation between amount of practice and number
of errors in tennis?

A. The increase in the amount of practices does not at all affect the
number of errors
B. As the amount of practice increases, the
number of errors decreases
757. What is the normal curve in the grade distribution of students in a class?

A. All have average grades


B. Most get average grades, few high and low

758. What can be said of student performance in a negatively skewed class score
distribution?

A. Most students performed well


B. Most students performed poorly

759. Which is true when the standard deviation is big?

A. Scores are spread out


B. Scores are concentrated

760. Which of the following best describes creativity?

A. Imaginative B. Positive
761. Of the types of validity for tests, what reflects the knowledge and skills in the program
of studies?

A. Curricular validity
B. Criterion validity

762. Of the types of validity for tests, what type constructs a test for a particular subject, the
items adequately reflecting the specific matter of the subject?

A. Content validity
B. Criterion validity

763. Principal Balita introduced machine-scored tests in his school. What factor in scoring does
this add to make tests more reliable?

A. Objective scoring
B. Ease in checking
764. Since Teacher Alvin wants his test be more reliable, which of the following should he
adopt?

A. Make a test that requires students the need for more time to complete
B. Increase number of test items

765. For easier correction of his test, Teacher Edward prepared the True and False items in
alternating 1, 3, 5, 7 etc. True, and 2, 4, 6 etc. False. What is wrong with the test in terms of
Usability?

A. Improper arrangement of items


B. Identifiable pattern of answers

766. What is the graphic illustration for the relationship between two variables?

A. Histogram B. Normal curves

767. For assessment in Problem-Based Learning, how do you classify the teacher’s
assessment of individual students presenting their output in class?

A. Peer assessment
B. Individual assessment

768. In assessment for Problem-Based Learning, who are responsible in judging student
performance?

A. Teacher and peers


B. Teacher-and-students

769. What is a scoring guide that lists the criteria and their levels of quality on which evaluation
will be focused?
A. Portfolio
B. Rubric

770. What is being assessed by the Rubric that seeks to assess high, moderate, or low
imaginative thinking?

A. Creativity
B. Craftsmanship
771. Using Rubrics to assess power-point presentations of students, which criteria from the
following is most significant as it is the essence of presentations?

A. Presenting design
B. Content value

772. Using Rubrics, which is futuristic and hypothetical among the following criteria chosen to
assess a planned student affairs program for the school year?

A. Projected impact of projects/activities


B. Feasibility of projects/activities

773. What should be the basis of remedial instruction?

A. Errors, fallings, and lapses


B. Identified systematic errors by students

774. Which of the following is the most reliable tool for determining the students’ ability to
write?

A. Portfolio assessment B. Scoring rubric

775. Which is the research method or technique suited for the study of Garbage collection in
NCR district I?

A. Focus group
B. Observational technique

776. What dependent variable can be used to test the hypothesis, “The more a teacher knows
about a specific subject matter, the better she can teach it”?

A. Teacher’s yearly performance rating


B. Personality traits of the teacher

777. For group guidance in classroom management, what element is lacking when there is
too much competitiveness and exclusiveness with the teacher being punitive and partial
to some students?

A. Poor interpersonal relations


B. Disturbance in group climate
105
778. Which of the following is the best option to deal with chronic behavior problems in the
classroom?

A. Indicate consequences of behavior


B. Establish positive expectations for students

779. Among styles of classroom management, which expects teachers to specify rules of
behavior, consequence for disobeying them, and to communicate these rules and
consequences clearly?

A. Assertive approach
B. Behavioral modification style

780. To manage behavior, the teacher needs to be able to identify the mistaken goals of
students who refuse to cooperate or participate?

A. Goal is to get revenge


B. Goal is to Isolate self

781. Of the approaches to classroom management, which emphasizes the organization and
management of students as they engage in academic work?

A. Group managerial approach


B. Business-academic approach

782. Teacher Ann Patuan dealt effectively with a minor infraction of whispering by a
student to a neighbor during class. Which of the following did she do?

A. Continue to teach and ignore infraction


B. Use nonverbal signals (gesture or facial
expression)

783. Teacher Dra D Explorer is a great lecturer and so she is invited to speak and represent the
school on many occasions. What is one quality of her lecturers when she follows a planned
sequence, not diverting so as to lose attention of her listeners?

A. Continuity
B. Fluency

106
784. If threat of punishment is necessary on erring students, how should this best be done?

A. Warn and threat at the same time


B. First a warning before the threat

785. Judging the relative severity of common classroom management strategies, which of
the following is least severe if applied on erring students?

A. Surprise quiz
B. Lowering school mark

786. For problems of discipline that cannot be resolved in class, which of the following should
not be done as it reduces the authority of the teacher?

A. Rely on others to resolve your classroom problems


B. Give students a chance to redeem himself

787. What mistake is teacher Benny trying to avoid when he never drops a topic before it is
completed?

A. Dangling topic B. Bounded topic

788. From classroom management strategies applied on erring students, which of the
following should not be done?

A. Parent-principal conference
B. Shaming erring student before the class

789. As preventive measure for classroom discipline, the teacher may restructure the
program. How is this not done?

A. Remove tension level before processing with lesson


B. Skip whole lesson unit altogether

790. Teacher Jose talks to students about their interests, what they did over the weekend,
their progress in school work, etc. What positive approach to classroom management did
Teacher Jose apply?

A. Being fair and consistent


B. Expressing interest for students

107
791. Of the approaches to classroom management, which involves a variety of techniques and
methods of ranging simple rewards to elaborate reinforcement, e.g. praise or rewards?

A. Behavioral modification approach


B. Assertive approach

792. To manage behavior, the teacher needs to be able to identify the mistaken goals of
students. What is the hidden goal of students who become violent?

A. Goal is to get attention


B. Goal is to get revenge
793. The class has become uncontrollable, and so teacher Iah decided to have some
undisciplined students out of the room to the counselor’s office. What did teacher Iah apply
by way of preventive approach to discipline?

A. Removing seductive objects


B. Antiseptic bouncing

794. To what problem type do students belong, who have difficulty due to low potential or lack
of readiness rather than motivation to work?

A. Underachiever
B. Low achiever

795. What is the term for the leap from theory to practice in which the teacher applies
theories to effective teaching methods and theories?

A. Integration process
B. Conceptualization process

796. Of subcategories of teacher movement behavior, what is happening when the teacher is
too immersed in a small group of students or activity, thus ignoring other students or
activity?

A. Truncation
B. Stimulus-bounded
797. Of subcategories of teacher movement behavior, what is happening when the teacher
bursts into activities without assessing student readiness and giver orders,
statements, or questions that confuse students?

A. Truncation B. Thrust

798. According to Nagel’s Acronym PPFF, what is important to Follow Up in the proper use of
instructional materials?

A. Student interest
B. Student expectations

799. Among unit plan components, to what cluster do projected competencies-


outcomes belong?

A. Objectives
B. Evaluation procedures

800. Of the following, which is not an example of instructional goals?

A. Critical understanding of Lyric poetic lines


B. Appreciation of art

801. Under the psychomotor domain of learning, what relates to expressive movement,
posture, gesture, facial expression and creative movements, e.g. taking part in a
play?

A. Non-discursive communication
B. Reflex movements

802. Under the domains of learning, to what domain do comprehension, analysis, and
evaluation belong?

A. Reflective B. Cognitive

803. Under the affective domain of learning, what includes active attention to stimuli such as
acquiescence, willingness to answer, and feelings of satisfaction?

A. Valuing B. Responding
804. Among cognitive objectives, what refers to the transfer of knowledge from once concept to
another?

A. Application B. Analysis

805. Of components of direct instruction, which involves teachers and students working
together on a skill or task and figuring out how to apply the strategy?

A. Consolidation B. Guided practice

806. Which of the following is an active strategy in transformative education?

A. Brainstorming B. Demonstration

807. Which of the following will not serve the aims of Sequential Synthesizing
Technique (SST) to enhance one-way lecture in class?

A. Plus-minus survey sheet


B. Small group discussion

808. Following a model procedure how is the process in Problem-Based learning sequenced?
I. List action for what we need to know
II. Explore the issues
III. Write solution with supporting document
IV. Develop and write problems statement

A. II, IV, I, and III B. III, I, IV, and II

809. Among examples listed, which does not represent teaching strategies to develop
metacognition?

A. Have the students develop questions about what is going on around them
B. Have teachers the free hand to monitor and rate learning
achievement
810. Of the following questions, which relate to metacognition?

A. How is this done?


B. Am I learning well enough?
811. Among teaching styles, how do you classify a teacher who clearly and fairly
communicates standards for discipline and performance to student?

A. Permissive B. Authoritative
812. How can the teacher best begin in developing a lesson plan design?

A. General learning goals B. Clearly defined outcomes 813. In the school,

who is the key to the success or failure in teaching-and-


learning, being the link between the child and the curriculum?

A. The teacher B. The parent

814. How do you characterize education that “shifts away” and introduces something new from
current educational practices?

A. Instructive B. Constructive

815. In the implementation of the curriculum at the classroom level, ineffective strategies
are called “Red”. Which of the following belongs to the Red Flag?

A. Content applied to real-life situations


B. Overemphasis on drill and practice

816. Which of the following has greatly influenced the introduction of K- to-12 reform in basic
education?

A. Need to conform international standards


B. Need to upgrade facilities of schools

817. What does the traditional curriculum and mass education lack that the progressive
curriculum can deliver?

A. Authoritarian role of the teacher


B. Value for individualization

818. Which of the following is the most appropriate use of television in the classroom?

A. TV viewing to replace absent teacher


B. TV viewing with cuts and explanation

111
819. While there is modernity and progress through the use of technology in instruction, what
is the greatest danger along the domain of learning that is posed by technology?
A. Developing passivity and uncritical minds
B. Developing visual and audial orientation

820. What is aroused among viewing learners by dramatic educational films in what is known
as “cliff hangers”?

A. Satisfaction B. Expectancy

821. In recent curricular revisions, the humanities are gradually replaced by technical courses
that bear practical consequences. This shows that curricular direction is toward .

A. Pragmatism B. Utilitarianism

822. According to the existentialist, every person is in the same predicament and has
the same possibilities. What does this imply?

A. Every person must go through the same form of education.


B. Every person must be given access to
education

823. School B’s curriculum puts emphasis on the classics. School B’s curriculum is
predominantly .

A. Perennialist B. Progressivist

824. Which teaching activity is founded on Bandura’s social learning theory?

A. Questioning B. Modeling

825. The practice of starting school programs very late points to the Filipinos’s .

A. Orientation only to the present


B. Lack of discipline

112
826. Teacher Cris always checks on entry knowledge and skills before she proceeds to her
new lesson. On which principle is Teacher Cris’ practice grounded?

A. Learning increases when the lesson is relevant.


B. New learning builds on previous learning.

827. Which practice is an offshoot of B. F. Skinner’s theory of operant conditioning?

A. Use of programmed instruction


B. Use of scaffolding
828. Student A claims: “I cannot see it so I won’t believe it.”Under which group can he be
classified?

A. Realist B. Empiricist

829. Teacher E encourages her students to develop their reasoning power to the fullest in
order to arrive at knowledge. Teacher E is a/an .

A. Pragmatist B. Rationalist

830. “Specialization is knowing more and more about less and less. Then it is better to be a
generalist”, claims Teacher Donald. On which philosophy does Teacher Donald
lean?

A. Essentialism B. Progressivism

831. “The greatest happiness lies in the contemplative use of the mind”, said Plato. Therefore,
let us give more opportunities for our students to do .

A. cooperative learning B. introspection

832. When we convince our students that pleasure is NOT the only good in life, we object to the
teachings of .

A. Stoicism B. Epicureanism

833. Teacher Michael teaches his/her students that pleasure is NOT the highest good. What
teacher teaches is contrary to which philosophy?

A. Hedonism B. Idealism
113
834. Class schedule, list of competencies to master, and uniform class requirements
belong to a school governed by the educational philosophy of .

A. Perennialism B. Essentialism

835. An emphasis on the humanities is a reflection of the philosophy.

A. Essentialist B. Perennialist

836. Which practice does NOT fit in a classroom that recognizes individual differences?

A. Sharing from multiple perspectives


B. Uniform requirements

837. The practice of motivating students before we proceed to our lesson proper is in
accordance with the law of .

A. Effect B. Readiness
838. If a student is encouraged to develop himself to the fullest and must satisfy his hierarchy
of needs, the highest need is .

A. Belongingness
B. Self-actualization

839. In a research conducted by Sadker. It was found out that male and female teachers are
more likely to call on male than female students and are more likely to give positive
reinorcement to male’s correct responses than to those of females. What does this
point to? Prevalence of in schools.

A. Racial prejudice
B. Gender bias

840. Teacher Lexus raises his eyebrows and tilts his head to communicate a warning to Janus
who talks while he lectures. Which classroom control technique did he use?

A. Prompting B. Eye contact


841. When asked about her order in a restaurant, a little girl tells the waiter, “yong kagaya
kahapon.” With Piaget’s theory in mind, what is the little girl’s behavior called?

A. Pre-operational egocentrism
B. Transductive reasoning

842. Fully convinced that it is not the right thing to do, Michael convinces his classmates not to
cut classes. In which stage of moral development is Michael?

A. Post-conventional
B. Pre-conventional

843. Which seat arrangement has been proven to be effective for learning?

A. Any seat arrangement to suit varied learing styles


B. A combination of fixed and flexible arrangement

844. Here is a lesson objective in Chemistry: “At the end of a 60-minute period, the students
should be able to memorize the periodic table of elements with 90% accuracy. “Applying
Robert Mager’s principle on writing performance objectives, which is the criterion
measure?

A. 90 % accuracy
B. memorize the periodic table

845. Which is in accordance with the “with-it-ness” principle of classroom management of


Kounin?

A. Teacher is fully aware of what is happening in his classroom.


B. Student is with his teacher in everything he teaches.
846. If teacher is concerned with the development of students’higher- order-thinking skills,
then his lesson objectives must go beyond
.

A. Comprehension
B. Application
847. When should a teacher decide to use direct instruction as a method?

A. When the subject matter is easy


B. When the subject matter is difficult

848. Although learning can take place anywhere and anytime, the more systematic the
teacher, the greater is the probability for success. This points out the need for teachers to
.

A. Select their instructional materials


B. Plan their lesson

849. With Bloom’s cognitive taxonomy in mind, which objective is in the highest level?

A. To rate a project along relevance, originally, and


craftmanship
B. To state the assumption that underlies the given statements

850. The following terms refer to performance objectives EXCEPT


objectives.

A. Teacher’s B. Learner’s

851. Edgar Dale found out that people generally remember 10% of what they read but
generally remember 90% of what they do. What does this imply to a teacher’s
instructional planning?

A. Plan a detailed lecture and provide the students with your lecture outline.
B. Plan for an activity where students do the real thing.

852. Which statement on review is CORRECT?

A. One of its purposes is to link the past with the new lesson.
B. It is repeating the lesson on the previous day to find out if students learned the
lesson.

853. Which objective in the affective domain is in the lowest level?

A. To accumulate examples of authenticity


B. To support viewpoints against abortion
854. Based on Erikson’s theory, what is the greatest danger in the elementary school
years for children’s development? The development of a sense of .
A. Isolation B. Inferiority

855. According to the cognitivist Jerome Bruner, for the learner to use information effectively,
it must be TRANSLATED into his TERMS. What is an implication of this to the teacher?

A. To make him define the terms clearly


B. To use terms within the level of the pupil’s
understanding

856. Aeneas, a Grade I pupil, plays with his classmates but cannot accept defeat. Based on
Piaget’s theory on cognitive development, in what development stage is Aeneas?

A. Sensorimotor
B. Pre-operational

857. Which learning principle is the essence of Gardner’s theory on Multiple Intelligence?

A. Intelligence is not measured in one form.


B. Learners have different IQ level.

858. Sandream, a Grade I pupil, is happy when he wins a game but sulks when he doesn’t.
Which does Sandream’s behavior indicate?

A. Egocentrism B. Egotism 859. Which

statement on IQ and EQ is correct?

A. It is the blending of both IQ and EQ that can make a difference


in performance.
B. The contribution of IQ and EQ to performance is dependent on factors like age
and gender.
860. Which is the main purpose of the compulsory study of the Philippine Constitution in
schools?

A. Develop students into responsible, thinking


citizens
B. Equip students with the basic knowledge of the Phil. Constitution

861. To develop in the young a sense of pride and appreciation of our very own culture,which
one should schools do more?
I. Stress on the positive aspects of our Filipino culture.
II. Invite knowlegdeable persons to discuss
events in our history for which we can be proud
III. Study the lives of our heroes.

A. I, II and III B. II and III

862. Based on Freud’s psychoanalytic theory, which component/s of personality is/are


concerned with a sense of right and wrong?
A. Ego B. Super ego

863. Which teaching practice is contrary to the child’s multiple intelligence theory of Gardner?

A. Using portfolio as an alternative way of assessing learning


B. Limiting the assessment of learning to paper-and-pencil tests

864. Baleleng says: “I must not cheat even if everyone in class cheats. Never mind if I get
ostracized for that. I will only cheat myself if I do. Besides, cheating erodes people’s
trusts and relationships. I condemn cheating.”In the context of Kohlberg’s moral
development theory, in what moral development level is Baleleng?

A. Post-conventional
B. Conventional

865. The following are characteristics of a child-friendly school EXCEPT


.

A. Not discriminating
B. Exclusive
866. Which does basic education encompass?
I. Early chilhood education
II. Elementary education
III. Secondary education
IV. Alternative learningsystem
V. Education for Children with Special Needs

A. I, II, III, IV and V


B. I, II, III and IV

867. To which goals of educational institutions as provided for by the Constitution is the
development of work skills attuned?

A. To develop vocational efficiency


B. To teach the duties of citizenship

868. Are elementary and secondary education compulsory for all children of schoolage?

A. No, this violates parents’and children ‘s right to decide what is best for
them and their children.
B. No, only elementary education is compulsory.

869. The Continuous Progression Scheme (CPS) in the elementary level in the Philippines in
the past had as its objective .

A. Equitable access to education


B. Mass promotion

870. Which program was wrongly understood to mean all pupils pass,no one fails?
A. Mastery Learning
B. Continuous Progression Scheme

871. Which move liberalized access to education during the Spanish period?

A. The provision of vocational training for school age children


B. The establishment of at least one primary school for boys and
girls in each municipality
872. Which is/are characteristic feature/s of the 2002 Basic Education Curriculum?
I. Constructivist
II. Integrative
III. Essentialist

A. I and II B. I,II and III

873. According to psychologist Ormrod, considering socio-cultural influences, who


can serve an effective model for students’motivation? One who is .

A. Not like them in terms of race, socio-economic status and cultural background
B. Similar to them in terms of race,socio- economic status and cutural background
874. Which program of the Department of Education is an offshoot of the Filipino bayanihan
spirit?

A. Child-Friendly School System


B. Brigada Eskwela

875. Which is a classroom application of this principle “Students learn more effectively when
they elaborate on new information” . Ask your students to .

A. Commit the principle to memory


B. Identify an application of the principle

876. Teacher Aljon gives his students opportunities to be creative because of his conviction
that much learning results from the need to express creativity. On which theory is Teacher
Aljon’s conviction anchored?

A. Behaviorist B. Humanist

877. Pavlov is to classical conditioning as is to operant conditioning.

A. B. F. Skinner B. J. Holt
878. Why is it sound to encourage our students to define terms in their own words? Because
.

A. students remember information better when they mentally process it in some


way.
B. they ought to connect the terms that they learn with other terms
879. Teacher Aldrex says: “If it is billiard that brings students out of the classroom, let us bring
it in the classroom. Perhaps, I can use it to teach Math.”To which philosophy does
Teacher Aldrex adhere?

A. Reconstructionism B. Progressivism 880. Which

illustrates a proactive approach to discipline?

A. Teacher starts giving rule only after a violation of a rule.


B. Teacher arranges the chairs to allow for a smooth traffic flow.

881. With preventive discipline in mind, which does NOT belong to the group?

A. Reactive B. Proactive 882. Which is

a proactive management practice?

A. Tell them that you enforce the rules on everyone, no


exception.
B. Set and clarify your rules and expectations on Day 1.

883. Teacher Liam strives to draw participation of every student into her classroom discussion.
Which student need is she trying to address? The need to .

A. show their abilities to the rest of the class


B. feel significant and be part of a group

884. Which can run counter to the encouragement you give to your students to ask questions?

A. Knitted eyebrows when a question is raised


B. Eye to eye contact

121
885. To encourage class participation, which ahould a teacher NEVER do?

A. Pose high level questions.


B. Reject a student’s response at once

886. Which question demands the highest level of thinking?

A. What conditions must be met for reporting method to be effective


B. Was the student reporting well done?
Support your answer?

887. Teacher Dang asked her class this question: How will you illustrate symbiosis to your
classmates? Teacher Dang’S question is a question that .
A. Assesses cognition
B. Probes creative thinking

888. Which questioning technique promotes more classroom interaction?

A. Focusing on convergent questions


B. Focusing on divergent questions

889. At what historical phase did Filipinos get formal tutoring for democratic sovereign life
which we started to enjoy in July 4, 1946?

A. June 12, 1898 declaration


B. American colonial regime

890. What established formal public school in the Philippines?

A. A Hispanic royal decree of 1863


B. Efforts of Spanish missionaries

891. Noypi is an Existentialism teacher. He made efforts to make connections


between new information of the EDSA peaceful revolution with past lessons on the
Philippine Hispanic revolution. How is this effort of teacher Noypi known in the cognitivist
learning theory?

A. Integration B. Modification
892. If Teacher Fin Gear sees honestly as a timeless virtue for yesterday/ today/and tomorrow,
what principle does she value?

A. Perennialism B. Pragmatism

893. Principal Noy Calbo is carefully focused on providing healthy, beautiful, and
nurturing physical and social environment in her school. She believes learners are
shaped by their own environment. What philosophical heritage of learning does she
value most?

A. Behaviorist B. Parennialist

894. What is stressed by the Reconstructionist philosophy in education?

A. Educational trends and issues


B. universal truths

895. In responding to the learning needs of special children, what negative value or condition
is directly remedied by our democratic educational system?

A. Elitism in education
B. discrimination among learners

896. In the Code of Ethics of Professional Teacher, moral values are given importance. What
aspect of moral character best relates to the teacher who required good habits and
attitudes while practicing them consistently in life?

A. being morally upright


B. being a loving person

897. What is the most basic foundation for moral principles?

A. The civil code


B. The natural law

898. Teacher Atejay is aware she cannot teach reading in a flash. Following the Goddle’s
Reading Skills Ladder, what step will she first focus on for beginners?

A. Using contextual clues. B. Basic sight words.


899. Which of the following is an example of non-verbal behavior by teacher?

A. Voice volume shows arrogance


B. Stands or sits that shows alertness

900. In the teaching career, which forms the essence of professional competence,
expertise and consciousness among teachers?

A. High position and ranking


B. Excellence in the practice of teaching

901. Teacher Jaja was awarded for her special efforts to create a nurturing environment in the
school. What character has she shown?

A. Humble
B. Loving

902. Teacher Petra thinks she is the best among peers, while others are below her stature.
What is her attitudes towards people?

A. “Bukas na lang” attitude


B. Holier than thou attitude

903. Which of the following can be a more effective strategy to advance higher esteem for the
teaching profession and its rightful share of available talents?

A. Removal of entry examinations for aspirants


B. Adequate remuneration and other means job
satisfaction

904. Public school teachers plan to organize a peaceful rally to express their grievances over
education policies. How can this be legally done?

A. Support of parents and community


B. Get permit for rally outside of work hours

905. Which of the following will most effectively encourage the new teachers for active
participation in the school unique educational culture?

A. Teacher induction program (TIP)


B. Involvement in planning/designing activities

124
906. What initiative provides unity and camaraderie among professional teachers?

A. Communication with former classmates


B. Membership in the teacher association

907. Schools are referred to as second homes. What pillar stands for caring and nurturing
schools?

A. Learning to be
B. Learning to live together

908. Teacher Tal Tal assert that the aim of development is the complete fulfillment of man
and his development in a holistic way as an individual, member of a family and
community and as a responsible citizen. Which fundamental type of learning does a
Teacher Tal Tal assert?

A. Learning to be
B. Learning to live together

909. What is the introduction of new ideas, goods, services, and practices in education e.g. new
educational technologies?

A. Progression B. Innovation

910. In his class, teacher Ta Hwan carefully focuses on important elements of the rich life of
indigenous communities, filtering out their negative attitudes. What attitudes among
diverse learners can this generate?

A. Sense of pride
B. Sense of apathy

911. Which of the following can be classified as a terminal learning outcome?

A. Sense of scientific attitude


B. Dichotomy between aquatic and land animals

912. Among unit plan components, to what cluster do models, replicas, charts, graphs and
specimens belong?

A. Resources and materials


B. Learning activities
125
913. Scientific attitudes need to be developed among young learners. Which does pupil
Fanny lacks when she accused without evidence her seatmate, Esonsa of stealing her
sandwich?

A. Persistence B. Objectivity

914. In today’s technology-society, what competency is gained by early learners in the use of
digital resources, such as electronic devices?

A. Social maturity B. computer literacy

915. Our digital generation reads from the internet. What kind of reading activity do we engage
in as we read and share Facebook materials?

A. Economic reading B. social reading

916. A schema is a computer chip in your brain that holds all you know on a subject. Each time
you find something new, you pull out the computer chip on that subject; add the new
information, and the return the chip to the storage. The reading technique that activates the
schema is:

A. Key-word reading B. Previewing

917. Michael Harrington noted: “If there is technological advance without social advance, there
is, almost automatically, an increase in human misery.” Which of the following shows Mr.
Harrington is right?

A. Internet Pornography
B. Facebook

918. Which of the following situation shows technology engaging learners in active,
constructive, intentional, authentic and cooperative learning?

A. Surfing Internet information sites


B. Collating materials from the Web

919. A young mother observes her seven year old girl glued to her computer games. What
aspect of family life may suffer due to obsession of the young with technology gadgets?

A. Family economic life


B. Parent-child relationship

126
920. In science teaching, what can be classified as self-directed strategy in knowing concepts,
principles and innovations?

A. Project B. Role playing

921. Teacher Bekang makes her preschoolers touch, rub and manipulate natural and
artificial objects. What is given importance in this case?

A. Tactile B. Anatomic

922. Teacher Empress Ki asked her students to bring small plants in pots, and then organize
and mark them according to categories, e.g. flowering, ornamental, herbal, etc. What
skill was desired in this activity?

A. Processing activity
B. Classifying activity

923. For young Filipino learners studying English as a second language, which of the
following can best generate language learning?

A. Simple exercise by use


B. Study of grammar

924. What can teacher Wan Lou encourage in order foster collaborative/ conversational
learning among her high school students?

A. Seek ideas and opinions from others (peers,


parents, friends, etc.)
B. Try-out ideas suggested by the teacher

925. Problems of discipline (misdeeds, lapses, minor offenses) can be reduced through
enthusiasm which can be matched by the enthusiasm of learners. What can draw a laugh
and reduce tension from all?

A. Nonverbal gestures
B. Sense of humor

926. What does a teacher do if she practices differentiated instruction?

A. Applies a multidisciplinary teaching approach


B. Makes students collaborate with other on instructional tasks
927. Due to the risk in exposing children in some equipment and apparatus, including
chemical elements in a science laboratory, what is an effective alternative teaching
strategy?

A. Lecture B. Demonstration

928. In the scientific process, what thinking skill is referred in the process of mentally analyzing
and evaluating data on evidences?

A. Constructive thinking B. Interpretative thinking 929. At what point does

the science learner engage in critical thinking?

A. Gathering evidence B. Examining or judging


930. Following Albert Bandura’s social learning theory, what can be achieved from the
public television live coverage of Senate hearings on plunder cases, such as on the
misuse of pork barrel funds?

A. Personality imaging B. Behavior change

931. Who among the educational psychologist gave importance to Eugenics or breeding,
stating that well-bred chicken create better, more moral and happier learning
environment?

A. William James B. Edward Thorndike

932. Among the following contemporary theories of learning, which explore situations
confined to certain people, places or events?

A. Cased-based reasoning
B. Constructivism

933. For the theorist of constructivism what/who is at the core of genuine thinking,
understanding and learning?

A. Learners B. Teachers

934. Beyond formal learning, modern school gives regard to parents and the community. What
contemporary theory of learning favors this attitude?

A. Social cognition B. Economic cognition


935. English teacher An Pituan teaches grammar by limited concentration on memory and
application of rules, immediately tested by daily quizzes. What learning theory is
applicable in this case?

A. Cognitivist B. Connectionist

936. Who introduce Gestalt psychology which influenced the importance of understanding in
the teaching-learning process?

A. Robert Gagne B. Max Wertheimer

937. Which of the following is NOT among the elements to consider in applying David
Ausubel’s advance organizer process?

A. Degree of prior familiarity


B. Subject overview

938. Following Ausubel’s social learning theory, what kind of behavior model is applied by
the Cinderella character in portraying moral lessons on life from rags-to-riches?

A. Symbolic B. Real
939. Within the context of constructive learning, which of the following manifests the teacher
as an effective teacher?

A. Great impression that teacher is technology savy


B. Discovery by students of learning opportunities

940. During Hispanic days in the Philippines, friars taught the Caratilia and discipline by the
use of the rod, on erring young “Indio” students. What type of conditioning was applied
for his practice?

A. Operant conditioning
B. Outright conditioning

941. It is believed that the teacher has an invisible aura of light surrounding his/her person that
influences the class. To what learning principle does this influence best relate to?

A. Learning by trial and error


B. Learning by observation and imitation

129
942. Learning is influenced by social interaction and interpersonal relations.
What must a teacher do?

A. Make students feel good about themselves


B. Make students work collaboratively

943. This is the scenario in Teacher Susan ’s class. The bell rang but the students are still
absorbed in their group discussion. The small groups haven’t move from their seats as
they are busy arguing on what the Philippine government should do with China’s
encroachment of the Scarborough shores. Which is true of the class?
I. Active learners
II. Does things for the sake of grades
III. They have strong knowledge base on government and international
relations

A. I,II and III B. I and III

944. Erickson viewed the elementary school years as critical for the development of self-
confidence. Which one should the elementary school provide?

A. Opportunities for children to achieve the recognition of teachers, parents


and peers by producing the things
B. Rules on academic behavior to be strictly enforced on all pupils

945. At the elementary level which must be done to curb pupil’s lack of motivation, low self-
esteem and lethargy?
I. Help children discover their special talents and interest
II. Organize interest clubs
III. Focus solely on academics for lesson mastery
A. I and II B. II only

946. There are values in the home environment that should form part of the school
environment during early childhood development. Which of the following has greatest
significance for carrying over form the home to the school?

A. Hygiene B. Discipline
947. Principal Ejay invited an expert to speak and answer questions on the issue of cybercrime
among faculty and senior students. What kind of discussion was held by the school?

A. panel discussion
B. Symposium discussion

948. Facilities such as classrooms, fixtures and equipment can often damage the morale of new
teachers and become an obstacle for adapting well to the school environment. What
should be the policy for assigning said physical facilities?

A. First come, first served basis


B. needs of student’s basis

949. Student Teacher Sophie has a very strong knowledge base in earth science. In effect,
which will she most likely be capable of doing?

A. Can make an experimental set up on the planets


B. Can answer all questions pertaining to earth science

950. What is teacher Noni’s teaching style which asserts his authority, and provides rewards
or punishment for work done?

A. Apprenticeship
B. Boss Employee

951. What kind of classroom may have harmful effects to preschoolers along academic
orientation or structures?

A. Adaption to age, maturity, cultural capabilities


B. Highly academic direct instruction

952. What makes assessment of learning peculiar is that it measures concepts, ideas and
abstract which can be characterized as
.

A. Physical B. Intangibles
953. Teacher Alden is observed to be a bit aloof from the children of the Aetas. She justifies
her action by saying: “We are but human; we cannot like every pupil.”Is she acting
ethically?

A. Yes, persons have their own peculiarities and we expect that we cannot like
everybody in the same way that not everybody can like us.
B. No, under no circumstances shall a teacher be prejudiced against any
learner.

954. The teacher is obliged to ensure certain qualities in choice and use of assessment
instruments. What was not observed when teacher Emperatress gave a test that
included materials that were not covered by class lessons?

A. Validity B. Reliability

955. Teachers evaluate student’s achievements because .

A. They want to know how their student perceived their method of teaching
B. They want to know if they are able to reach the goal of their teaching efforts

956. Teacher Francis organized a structured class discussion with two opposing sides
and assigned speakers on the issue of contraceptives. What was this kind of class?

A. panel discussion B. Debate

957. On managing the class as a group, what is the problem when a teacher gives work load to
students that are too light or too heavy?

A. dissatisfaction
B. disturbance in group climate

958. Guided by Max Scheler’s hierarchy of values, what did student Ki- Seung Nyang prefer
to practice when she decided to go on an out-of- town picnic to enjoy the day with friends,
rather than attend a required Saturday class?

A. Vital values
B. Pleasure values
959. Of the following, which demonstrates the principle that the good end does not justify the
illegal/immoral means?

A. Students form a peer group to pass the course


B. A principal collects money from students to set up a computer laboratory

960. Which is considered ethical when it comes to a teacher applying for a teaching position?
I. There is a vacant position
II. She/he is qualified for the position
III. She/he respects the system of selection on the basis of merit and competence

A. I, II, and III B. I and II

961. How is the moral character of a teacher most accurately summed up when he/she have
reached a level of development emotionally, socially, mentally, spiritually, appropriate
to age and status?

A. Being loving person


B. Being fully human
962. Teacher Chang does not personally agree with one school policy. What is the professional
thing for her to do?

A. Make an honest effort to understand, support, and carry out the school policy
even if he does not personally agree.
B. Be indifferent about it as he exerts effort to understand.

963. Which of the following steps should be completed first in planning an achievement test?

A. Set up a table of specifications


B. Define the instructional objective

964. One every important type of communication skill which is receptive in nature and requires
one to be very alert is:

A. Reading B. Listening
965. Magda is already in College. What reading activities should she be guided on the
following stages/period of continuity/progression in developmental reading?

A. Reading readiness
B. Refinement in the use of reading materials

966. Teacher Baek An manages a Nursery class. What stage/period of reading should she
observe in teaching reading skills to her pupils?

A. Reading readiness
B. Beginning reading

967. Which of the following opinions about books is about the sensible delight they give the
reader?

A. A man without book is like a body without a soul


B. A book is a garden in pocket

968. Ranking of teacher applicants based on LET rating, college GPA, demo teaching
interview, etc. is in accordance with the hiring principle of
.

A. Selective admission to the profession


B. Aristocracy

969. Which is NOT a manifestation of a professional teacher’s pride in a teaching as a noble


calling?

A. She is resigned to the idea that teaching is for those who cannot make it in
the other profession.
B. She campaigns for the better students to take up teaching.

970. A teacher is a facilitator of learning and of the development of the youth. Which practice
is NOT in keeping with his role as facilitator?
A. Considers the multiple intelligences of learners
B. Humiliates misbehaving pupils
971. Teacher Purang does not want Teacher Pazang to be promoted and so the former
fabricates lies against Teacher Pazang and writes them in an anonymous letter mailed to
the Schools Division Superintendent. What should Teacher Purang do if she has to act
prefessionally?

A. Go to the Schools Division Superintendent and give criticism verbally.


B. Submit to the office concerned a signed justifiable criticism against Teacher
Pazang.

972. Under no circumstances shall a teacher be prejudiced nor discriminatory against


any learner, says the Code of Ethics. When is a teacher prejudiced against any learner?

A. When he makes a farsighted pupil sit at the back


B. When he refuses a pupil with a slight physical disability in class

973. Authority comes from God and is meant to .

A. Make the subjects of authority recognize their superiors


B. Help those given the authority do their task

974. In school-based management, school head is given more power for him .

A. Be able to serve his teachers, pupils and community better


B. Command his subordinates to do what he pleases 975. To insist

on rights without acceptance of duty is .

A. Selflessness B. Selfishness

976. Human dignity is inherent in every person. What does this mean? The inner worth of a person
.

A. Depends on what s/he can do


B. Is in him/her whether or not s/he can do or has something 977. Which

statement on human rights is CORRECT?

A. Every right carries with an obligation.


B. Rights are absolute.
978. Duties that secure public order or common good have priority over those that safeguard
the individual. Which illustrates this?

A. The need to obey traffic rules is prior to one’s personal convenience.


B. Drugstores require doctor’s prescription before disposing the drug.

979. If you agree with Rizal on how you can contribute to our country’s redemption, which
should you work for?
A. The economic recovery of the country
B. The upgrading of the quality of Philippine education

980. Which one should you do if a parent of one of your failing pupils asks you to tutor her
daughter in consideration of a certain amount of money which you badly needed?

A. Accept the offer but do tutoring outside office hours.


B. Advise the parent to look for another tutor.

981. Teacher Dadang is ordered to pass undeserving student with a death threat. Which
advice will a hedonist give?

A. Pass the student. That will be of use to the student, his parents and you.
B. Pass the student. Why suffer the threat?

982. In times of war, soldiers must be ready to give up their lives in defense of the nation.which
normative relationship applies this case?

A. More common good-public safety before private gain


B. Wider social order-the family before the individual

983. Which illustrates this principle: “Circumstances may change a good or an indifferent act
into a punishable one”.

A. The professional teacher’s act of insulting a student is worse than a student


belittling a classmate.
B. Slapping someone at a moment of rage is not as worse as slapping someone
for thrill.
984. The early Egyptians adopted apprenticeship as one educational method. Which is
its equivalent in our present teacher education curriculum?

A. Presentation and defense of a research paper


B. Practicum like practice teaching

985. Teacher Dante advises the class: “Better be good in this life. Treat others well. The good
that you do to others will come back to you in the same way that the bad you do to others
will boomerang to you.” This is what they call karma. From which thought did this
originate?

A. Buddhist B. Hindu

986. Jerome Bruner’s idea that interest in the material being learned was the best stimulus for
learning supports our efforts to develop motivation in our students.

A. Intrinsic B. Extrinsic

987. Helping in the development of graduates who are “maka-Diyos” is an influence of


morality.
A. Naturalistic
B. Classical Christian

988. With which philosopher did the emphasis on critical thinking begin?
This philosopher said “The unexamined life is not worth living.”

A. Pythagoras B. Socrates

989. Which classroom management technique is in line with Lao tsu’s thought on inaction?

A. Planned ignoring
B. Loss of privileges

990. Which is NOT true of the guidance process?

A. Guidance is meant for every student including the performing


students.
B. The guidance counselor decides what is best for the student
who is in the dark.
137
991. Self-rating and a follow-up conference with teacher benefit both teacher and student.
The following are benefits for the student EXCEPT

A. heightens student’s defensiveness.


B. helps the student develop increased skill in self-assessment.

992. Teacher Uber asked her pupils to create a story out of the given pictures. Which
projective technique did Teacher Uber use?

A. Rorschach test
B. Thematic Apperception Test

993. If a researcher commits a type 1 error, what does he commit?

A. Accepting the null hypothesis when it should havebeen rejected


B. Rejecting the null hypothesis when it should have been
accepted

994. Which is TRUE when standard deviation is small?

A. Scores are tightly bunched together.


B. The bell curve is relatively flat.

995. Principal announced this in a faculty meeting: “There is a positive correlation between
the students’ scores in Math and in Science. “Which is the CORRECT interpretation?

A. The students who scored high in Math also scored high in


Science.
B. The students who scored high in Math scored lowin Science.
996. Which CANNOT enhance the comparability of grades?

A. Using common conversion table for translating the test scores to ratings
B. Formulating tests that vary among teachers

997. Which is claimed to be a disadvantage of our current practice of giving numerical or letter
grade?

A. They are perceived to be absolute measures of intelligence.


B. They don’t give feedback on students’ specificstrengths and
weaknesses.
138
998. In the context of grading, what is referred to as teacher’s generosity error? A teacher
.

A. has a tendency to give high grades as compared to the rest


B. gives way to students’ bargain for no more quiz

999. Teacher Lorelie does norm-referenced interpretation of scores. Which of the following
does she do?

A. She compares individual student’s scores with other


students’scores.
B. She describes what should be every individual student’s performance.

1000. If the lowest score obtained in a 100-item test is 1 and the highest is 99, what can be said
roughly of the scores? The scores are .

A. highly similar
B. highly dispersed

PROFESSIONAL EDUCATION ANSWER KEYS


501 A 531 A 561 A 591 A 621 B
502 A 532 A 562 A 592 B 622 B
503 B 533 B 563 B 593 A 623 B
504 B 534 B 564 B 594 B 624 A
505 B 535 B 565 A 595 B 625 A
506 B 536 B 566 B 596 B 626 B
507 A 537 A 567 B 597 B 627 A
508 B 538 B 568 B 598 B 628 A
509 B 539 A 569 B 599 B 629 A
510 B 540 B 570 A 600 A 630 A
511 A 541 A 571 A 601 B 631 B
512 B 542 A 572 A 602 A 632 A
513 A 543 B 573 A 603 B 633 A
514 A 544 A 574 B 604 B 634 A
515 A 545 A 575 B 605 A 635 A
516 B 546 B 576 B 606 B 636 A
517 A 547 A 577 B 607 B 637 B
518 B 548 B 578 A 608 A 638 A
519 B 549 A 579 A 609 B 639 A
520 A 550 A 580 B 610 B 640 B
521 A 551 B 581 B 611 B 641 B
522 B 552 A 582 A 612 A 642 B
523 B 553 B 583 B 613 A 643 A
524 A 554 A 584 A 614 A 644 A
525 A 555 B 585 A 615 B 645 A
526 A 556 B 586 B 616 B 646 A
527 B 557 A 587 A 617 B 647 B
528 A 558 B 588 A 618 A 648 B
529 A 559 A 589 A 619 A 649 B
530 A 560 A 590 A 620 A 650 B
651 B 691 B 731 B 771 B 811 A
652 A 692 B 732 B 772 A 812 B
653 B 693 A 733 A 773 A 813 A
654 A 694 A 734 A 774 B 814 B
655 A 695 A 735 A 775 B 815 B
656 B 696 B 736 B 776 A 816 A
657 B 697 A 737 A 777 B 817 B
658 A 698 A 738 A 778 A 818 B
659 B 699 B 739 A 779 A 819 A
660 A 700 A 740 A 780 B 820 B
661 B 701 A 741 A 781 A 821 A
662 A 702 A 742 B 782 B 822 B
663 B 703 B 743 A 783 A 823 A
664 B 704 A 744 B 784 B 824 B
665 A 705 A 745 B 785 A 825 B
666 B 706 A 746 A 786 A 826 B
667 A 707 B 747 B 787 A 827 A
668 A 708 B 748 B 788 B 828 B
669 A 709 A 749 A 789 B 829 B
670 B 710 A 750 A 790 B 830 A
671 A 711 A 751 B 791 A 831 B
672 A 712 B 752 A 792 B 832 B
673 B 713 A 753 B 793 B 833 A
674 A 714 B 754 A 794 B 834 B
675 B 715 A 755 B 795 A 835 B
676 A 716 B 756 B 796 B 836 B
677 A 717 B 757 B 797 B 837 B
678 A 718 B 758 A 798 A 838 B
679 A 719 A 759 A 799 A 839 B
680 B 720 A 760 A 800 B 840 A
681 B 721 B 761 A 801 A 841 A
682 A 722 B 762 A 802 B 842 B
683 A 723 B 763 A 803 B 843 A
684 B 724 A 764 B 804 A 844 A
685 A 725 B 765 B 805 B 845 A
686 B 726 A 766 A 806 A 846 B
687 B 727 A 767 B 807 B 847 B
688 A 728 A 768 B 808 A 848 B
689 A 729 A 769 A 809 B 849 A
690 B 730 B 770 A 810 B 850 B
851 A 891 A 931 B 971 B
852 A 892 A 932 A 972 B
853 A 893 A 933 A 973 B
854 B 894 A 934 A 974 A
855 B 895 B 935 A 975 B
856 B 896 A 936 B 976 B
857 A 897 B 937 B 977 A
858 B 898 B 938 A 978 A
859 A 899 B 939 B 979 B
860 A 900 B 940 A 980 B
861 A 901 B 941 B 981 B
862 B 902 B 942 B 982 A
863 B 903 B 943 B 983 B
864 A 904 B 944 A 984 B
865 B 905 B 945 A 985 B
866 A 906 B 946 B 986 A
867 A 907 B 947 B 987 B
868 B 908 A 948 B 988 B
869 B 909 A 949 B 989 A
870 B 910 B 950 B 990 B
871 B 911 A 951 B 991 A
872 B 912 B 952 B 992 B
873 B 913 B 953 B 993 B
874 B 914 B 954 A 994 A
875 B 915 B 955 B 995 A
876 B 916 A 956 B 996 B
877 A 917 A 957 A 997 B
878 A 918 B 958 B 998 A
879 B 919 B 959 B 999 A
880 B 920 A 960 A 1000 B
881 A 921 A 961 B
882 B 922 B 962 B
883 B 923 A 963 A
884 A 924 A 964 B
885 B 925 B 965 B
886 B 926 A 966 A
887 B 927 B 967 B
888 B 928 B 968 A
889 B 929 B 969 A
890 A 930 A 970 B

S-ar putea să vă placă și